🔙 Quay lại trang tải sách pdf ebook Tạp Chí Epsilon Số 23 Ebooks Nhóm Zalo Tạp Chí online của cộng đồng những người yêu to án VỀ BÀI TOÁN CROATIA TST 2011 Nguyễn Song Minh ĐỒ THỊ CỦA HÀM SỐ ĐA THỨc Lê Phúc Lữ & Trần Nguyễn Thanh Danh THÁNG 6/2023 NO.21 GIỚI THIỆU ĐẶC TRƯNG EULER VÀ MỘT SỐ ỨNG DỤNG Trần Thị Ánh Dương Trần Quang Hùng MỘT VÀI TÍNH CHẤT HÌNH HỌC TRÊN MỘT CẤU HÌNH VỀ TỶ SỐ VÀNG Biên tập viên: Lê Viết Ân Võ Quốc Bá Cẩn Trần Quang Hùng Nguyễn Văn Huyện Lê Phúc Lữ Tống Hữu Nhân Nguyễn Tất Thu Trần Bình Thuận Đặng Nguyễn Đức Tiến Chủ biên: Trần Nam Dũng LỜI NGỎ Kính chào độc giả Epsilon, chào mùa hè 2023! Mùa hè là thời điểm tuyệt vời để nghỉ ngơi và tận hưởng cuộc sống. Đó là thời gian mà chúng ta có cơ hội khám phá và tìm hiểu những điều mới mẻ. Và đó cũng là thời khắc mà chúng tôi gửi đến Epsilon cho tất cả độc giả của mình. Trong số báo Epsilon 23 này, chúng tôi tự hào mang đến cho bạn những câu chuyện đặc biệt, những bài toán thú vị và những ý tưởng độc đáo. Bạn sẽ được khám phá những chủ đề như đặc trưng Euler và ứng dụng, sự đối lập giữa lớp văn hóa Ấn và lớp văn hóa Hán, cách tiếp cận tổng thể trong việc dạy Ngôn Ngữ và Toán, ý nghĩa của bất đối xứng, những tính chất hình học trên cấu hình Tỷ số Vàng và nhiều chủ đề hấp dẫn khác. Chúng tôi hy vọng rằng số báo Epsilon 23 sẽ mang đến cho bạn những trải nghiệm thú vị và kiến thức bổ ích trong mùa hè này. Xin chân thành cảm ơn sự ủng hộ và đồng hành của bạn! Chúc bạn có một mùa hè tràn đầy niềm vui và ý nghĩa! Trân trọng, Ban Biên tập Epsilon MỤC LỤC Trần Thị Ánh Dương Giới thiệu đặc trưng Euler và một số ứng dụng . . . . . . . . . . . . . . . . . . . . . . . . . . 5 Nguyễn Lê Anh Có hay không lớp văn hóa Ấn trước lớp văn hóa Hán? . . . . . . . . . . . . . . . . . . . . . . 27 Nguyễn Ái Việt Tiếp cận tổng thể tới dạy Ngôn Ngữ và Toán . . . . . . . . . . . . . . . . . . . . . . . . . . . 31 Trần Văn Trản Sự cần thiết của bất đối xứng . . . . . . . . . . . . . . . . . . . . . . . . . . . . . . . . . . . 35 Trần Quang Hùng Một vài tính chất hình học trên một cấu hình về Tỷ số Vàng . . . . . . . . . . . . . . . . . . . 39 Lê Phúc Lữ và Trần Nguyễn Thanh Danh Đồ thị của hàm số đa thức . . . . . . . . . . . . . . . . . . . . . . . . . . . . . . . . . . . . . 45 Đào Xuân Luyện Bài toán xây dựng đa thức . . . . . . . . . . . . . . . . . . . . . . . . . . . . . . . . . . . . . 58 Trần Nam Dũng Câu chuyện về định lý Trung Hoa về số dư . . . . . . . . . . . . . . . . . . . . . . . . . . . . 72 Nguyễn Song Minh Về bài toán Croatia TST 2011 . . . . . . . . . . . . . . . . . . . . . . . . . . . . . . . . . . . 83 Nguyễn Song Thiên Long Một số bổ đề về giới hạn của dãy số . . . . . . . . . . . . . . . . . . . . . . . . . . . . . . . 90 Tạ Duy Phượng và cộng sự Giới thiệu trò chơi tháp Hà Nội . . . . . . . . . . . . . . . . . . . . . . . . . . . . . . . . . . 114 4 Tạp chí Epsilon, Số 23, 06/2023 Giới thiệu đặc trưng Euler và một số ứng dụng Trần Thị Ánh Dương (Trường Phổ thông Trung học Lê Chân, Hải Phòng) Giới thiệu. Bài viết giới thiệu một số chứng minh công thức F − V + E = 2 (đặc trưng Euler) và một số ứng dụng của đặc trưng Euler. 5 Tạp chí Epsilon, Số 23, 06/2023 1. Mở đầu Đặc trưng Euler, hay công thức F − V + E = 2 là một trong 17 phương trình làm thay đổi thế giới (xem [1]). Do tính bản chất và quan trọng của công thức này, đặc trưng Euler có đến vài chục cách chứng minh (xem [7]) và có nhiều ứng dụng (xem, thí dụ, [6]). Đặc trưng Euler (còn được gọi là bất biến Euler, công thức Euler, hoặc đặc trưng Euler-Poincaré) là một bất biến tôpô, là số không đổi đặc trưng cho hình dạng hoặc cấu trúc của một không gian tôpô không phụ thuộc vào cách nó bị biến dạng. Đặc trưng Euler thường được ký hiệu là X . Đặc trưng Euler X (S) của một đa giác phẳng S được chia thành các tam giác bằng số đỉnh trừ đi số cạnh cộng với số mặt của tam giác trong đa giác đó: X (S) = V − E + F. Đa diện lồi bất kì cũng có đặc trưng X = V − E + F = 2, trong đó V , E và F tương ứng là số đỉnh (góc), các cạnh và mặt của khối đa diện. Kết quả này cũng còn được gọi là công thức đa diện Euler hoặc định lý đa diện Euler. Đặc trưng Euler đã được xác định cho các khối đa diện và được sử dụng để chứng minh định lý về số các khối đa diện đều khác nhau (phân loại các khối Platon). Leonhard Euler, tên của ông được đặt cho khái niệm này, đã có các công trình nghiên cứu đầu tiên về đặc trưng này. Ta có: Tên Đỉnh V Cạnh E Mặt F X = V − E + F Tứ diện 4 6 4 2 Hình lập phương 8 12 6 2 Bát diện 6 12 8 2 Thập nhị diện 20 30 12 2 Nhị thập diện 12 30 20 2 Hình 1. Các khối đa diện Platon. Ta cũng có thể mở rộng đặc trưng Euler (tức công thức X = 2) cho hình cầu và áp dụng cho các khối đa diện cầu. 6 Tạp chí Epsilon, Số 23, 06/2023 Bài viết này giới hạn trong phạm vi giới thiệu một số cách chứng minh cơ bản và một số ứng dụng tiêu biểu của đặc trưng Euler. 2. Một số chứng minh công thức đặc trưng Euler Mục này trình bày một số cách chứng minh công thức đặc trưng Euler. 2.1. Chứng minh dựa trên lý thuyết đồ thị Biểu diễn phẳng của một đồ thị chia mặt phẳng thành các miền, kể cả miền vô hạn. Ví dụ biểu diễn phẳng của đồ thị trên Hình 2 chia mặt phẳng thành 6 miền. Chúng được gán nhãn như hình vẽ. Euler đã tìm ra mối quan hệ giữa số miền, số đỉnh và số cạnh của một đồ thị phẳng. Ta có: Định lý 1 Nếu G là một đồ thị phẳng liên thông có V đỉnh, E cạnh và F miền thì V − E + F = 2. 2.1.1 Chứng minh 1 Trước tiên Xem thêm chứng minh ta xác định biểu diễn phẳng của G. Ta sẽ chứng minh định lí bằng cách xây dựng một dãy các đồ thị con G1, G2, ..., Ge = G, mỗi bước ghép thêm một cạnh vào đồ thị ở bước trước. Điều này làm được khi sử dụng phương pháp quy nạp toán học. Lấy tùy ý một cạnh của G để nhận được G1. Để nhận được Gn từ Gn−1 ta thêm tùy ý một cạnh liên thuộc với một đỉnh của Gn−1 và thêm một đỉnh khác liên thuộc với cạnh mới đó, nếu nó chưa có trong Gn−1. Điều này làm được vì G liên thông. G sẽ nhận được sau khi e cạnh được ghép thêm vào các đồ thị tạo ra trước. Gọi Rn, En và Vn tương ứng là số miền, số cạnh và số đỉnh của biểu diễn phẳng của Gn do biểu diễn phẳng của G sinh ra. Hệ thức R1 = E1 −V1 + 2 là đúng với G1 vì E1 = 1, V1 = 2 và R1 = 1. Bây giờ giả sử theo quy nạp rằng Rn = En − Vn + 2. Gọi {an+1, bn+1} là cạnh gộp vào Gn để được Gn+1. Có hai khả năng xảy ra. 7 này ở [2], trang 584. Tạp chí Epsilon, Số 23, 06/2023 Trường hợp 1. Cả hai đỉnh an+1, bn+1 đã thuộc Gn. Khi đó chúng phải ở trên biên của miền chung R nếu không thì không thể gộp cạnh {an+1, bn+1} vào Gn mà không có các cạnh cắt nhau (Gn+1 là phẳng). Cạnh mới này sẽ chia miền R thành hai miền con. Do đó Rn+1 = Rn + 1, En+1 = En + 1 và Vn+1 = Vn. Do vậy ta có công thức Rn+1 = En+1 − Vn+1 + 2. Trường hợp 2. Một trong hai đỉnh của cạnh mới chưa thuộc Gn. Giả sử an+1 thuộc Gn còn bn+1 không thuộc Gn. Thêm cạnh này không sinh ra một miền mới nào, vì bn+1 phải ở trong miền có an+1 ở trên biên của nó. Do đó, Rn+1 = Rn. Nhưng En+1 = En + 1 và Vn+1 = Vn + 1. Mỗi vế của công thức không đổi nên công thức vẫn đúng. Nói cách khác Rn+1 = En+1 − Vn+1 + 2. Vậy với mọi n ta đều có Rn = En − Vn + 2. Vì đồ thị gốc là Ge nhận được sau khi thêm e cạnh. Định lí được chứng minh. Công thức Euler được minh họa trong ví dụ sau: Ví dụ Giả sử đơn đồ thị phẳng liên thông có 20 đỉnh, mỗi đỉnh đều có bậc bằng 3. Biểu diễn phẳng của đồ thị này chia mặt phẳng thành bao nhiêu miền? Lời giải Đồ thị phẳng này có 20 đỉnh, mỗi đỉnh đều có bậc bằng 3, do vậy V = 20. Vì tổng số bậc của các đỉnh, 3V = 3.20 = 60, bằng hai lần số cạnh, tức là 2E, ta có E = 60 : 2 = 30. Do vậy theo công thức Euler, số các miền là: R = E − V + 2 = 30 − 20 + 2 = 12. 2.1.2 Chứng minh 2 Đồ thị Xem thêm chứng minh trong Hình 2 có năm đỉnh, bảy cạnh, bốn miền và 5 − 7 + 4 = 2. này ở [4], trang 120. Hình 2. Ví dụ một đồ thị có năm đỉnh, bảy cạnh và bốn miền. Nếu không tính vùng không giới hạn là miền thì công thức Euler trở thành V − E + F = 1. Xét một cây (một đồ thị phẳng liên thông và không có chu trình). Vì cây không có chu trình, miền duy nhất là miền không bị giới hạn, nên công thức Euler V − E + 1 = 2 hay V = E + 1. Do đó, số đỉnh của cây lớn hơn số cạnh 1 đơn vị. Ta dùng cách loại bỏ các cạnh ra khỏi đồ thị để chứng minh định lí. 8 Tạp chí Epsilon, Số 23, 06/2023 Xét một đồ thị phẳng liên thông. Chọn một cạnh bất kì. Cạnh có thể liên thuộc hai đỉnh hoặc là một khuyên. Hình 3. Loại bỏ cạnh a, c, d. Giả sử cạnh liên thuộc hai đỉnh. Ta thu nhỏ cạnh cho đến khi nó biến mất hoàn toàn và trở thành một đỉnh. Điều này có thể thực hiện trong đồ thị phẳng. (xem loại bỏ cạnh a, c, d trong Hình 3). Như vậy sẽ làm cho số cạnh và số đỉnh giảm đi 1 đơn vị. Số miền không đổi. Do đó, giá trị của biểu thức V − E + F không thay đổi. Giả sử cạnh là một khuyên. Ta loại bỏ các cạnh b, e. Do đó, số cạnh và số miền giảm đi 1 đơn vị. Số đỉnh không thay đổi. Vì vậy, giá trị của biểu thức V − E + F không thay đổi. Tiếp tục quá trình loại bỏ cạnh cho đến khi còn lại một đỉnh duy nhất, không có cạnh và có một miền (miền ngoài). Do đó, V − E + F = 2. Bởi vì V − E + F không đổi trong suốt quá trình nên V − E + F = 2 đúng với đồ thị ban đầu. 2.2. Phương pháp điện tích 2.2.1 Điện tích (Electrical Charge) Đặt khối Xem thêm chứng minh đa diện trong không gian sao cho không có cạnh nằm ngang. Khi đó, chỉ có duy nhất một đỉnh cao nhất U và một đỉnh thấp nhất L. Đặt một điện tích dương tại mỗi đỉnh, một điện âm tại chính giữa mỗi cạnh và một điện tích dương ở giữa mỗi mặt. Ta cần chỉ ra rằng, mọi điện tích đều bị khử, trừ hai điện tích tại U và L. Bỏ mọi điện tích ở đỉnh và cạnh vào mặt kế bên, sau đó nhóm hết điện tích trong mỗi mặt lại với nhau. Hướng di chuyển được xác định theo quy luật: mỗi điện tích di chuyển theo phương ngang, ngược chiều kim đồng hồ. Như vậy, mỗi mặt nhận một tổng điện tích từ khoảng không gian dọc theo giới hạn của nó. Không gian mở này luân phiên tách thành cạnh và đỉnh. Vì điện tích đầu tiên và cuối cùng là ở cạnh, sẽ có dư một điện tích âm. Cho nên tổng điện tích trong mỗi mặt đều bằng 0. Và tất cả chỉ còn lại + 2 cho đỉnh U và L. 2.2.2 Điện tích đối ngẫu (Dual Electrical Charge) này ở [7] Xoay khối đa diện sao cho không có cạnh nào nằm dọc. Xem thêm chứng minh này ở [7] 9 Tạp chí Epsilon, Số 23, 06/2023 Giống như phương pháp trước, đặt một điện tích dương ở mỗi đỉnh và giữa các mặt; điện tích âm ở chính giữa các cạnh. Ta cần chỉ ra rằng, tất cả mọi điện tích đều bị khử, trừ hai điện tích dương. Di chuyển điện tích trên mỗi cạnh đến điểm tận cùng bên phải của nó; di chuyển điện tích trên mỗi mặt (ngoại trừ mặt ngoài cùng) đến đỉnh gần nhất bên phải của nó. Mọi đỉnh (ngoại trừ đỉnh ngoài cùng bên trái) lần lượt nhận điện tích của các cạnh và mặt; triệt tiêu với điện tích ban đầu của nó. Chỉ còn lại duy nhất 2 điện tích dương ở mặt ngoài cùng và đỉnh ngoài cùng bên trái là chưa triệt tiêu. 2.3. Phương pháp sử dụng góc hình cầu (Spherical angles) Phương pháp này Xem thêm chứng minh sử dụng tổng các góc trong tam giác cầu trên mặt cầu. Chìa khóa để chứng minh của Legendre là một công thức từ hình học hình cầu cho diện tích tam giác trên bề mặt của quả cầu theo góc bên trong. Trên hình cầu, hình tam giác và các hình đa giác không được tạo thành từ các đoạn thẳng mà từ vòng cung của đường tròn lớn. Một đường tròn lớn là bất kì đường tròn nào trên quả cầu có bán kính tương đương với bán kính quả cầu. Chúng ta xác định một tam giác trên hình cầu được tạo thành bởi ba đường tròn lớn, được gọi là tam giác trắc địa (geodesic triangle). Nhiều định lí đối với tam giác phẳng cũng đúng đối với tam giác trắc địa, chẳng hạn: tổng hai cạnh của tam giác luôn lớn hơn cạnh còn lại. Nhưng có một tính chất không đúng. Đó là, trong hình học phẳng tổng các góc trong tam giác bằng 1800 nhưng trên mặt cầu, tổng các góc trong tam giác trắc địa luôn lớn hơn 1800. Vào thế kỉ 17, Thomas Harriot (1560-1621) và Albert Girard (1595-1632) đã chứng minh định lí sau. Định lý 2 (Định lí Harriot-Girard đối với tam giác) Tam giác trắc địa trên mặt cầu đơn vị với ba góc trong a, b, c có diện tích S = a + b + c − π. Bởi vì tổng các góc trong tam giác phẳng là π nên có thể viết lại công thức trên như sau: S = (a + b + c) - tổng các góc trong tam giác phẳng. Lời giải Giả sử hình cầu đơn vị có bán kính R = 1. Khi đó diện tích của nó là Smc = 4π. Ta sử dụng một vật được gọi là lune (lưỡi liềm). Lune là miền được giới hạn bởi hai đường tròn lớn. Hai đường tròn lớn luôn cắt nhau tại hai điểm đối xứng trên mặt cầu. 10 này ở [4], trang 89. Tạp chí Epsilon, Số 23, 06/2023 Nếu lưỡi liềm có một góc a thì góc đối diện cũng bằng a. Ta có: Smc=a2π. Slune Khi đó diện tích lưỡi liềm là Slune =a2πSmc =a2π4π = 2a. Xét một tam giác trắc địa ABC trên mặt cầu đơn vị, có các góc a, b, c. Tam giác nằm ở một nửa bán cầu. Mở rộng các cạnh của tam giác ABC cắt biên của bán cầu. Gọi D, E, F, G, H, I là các giao điểm. Theo tính chất đối xứng, SADE + SAGH = Slune = 2a. Tương tự, SBF G + SBDI = 2b; SCHI + SCEF = 2c. Suy ra (SADE + SAGH) + (SBF G + SBDI ) + (SCHI + SCEF ) = 2a + 2b + 2c. Do đó: 1 2Smc + 2SABC = 2a + 2b + 2c ⇒ 2π + 2SABC = 2a + 2b + 2c ⇒ SABC = a + b + c − π Định lý 3 (Định lí Harriot-Girard đối với đa giác) Diện tích của đa giác trắc địa(geodesic polygon) n - cạnh trên mặt cầu đơn vị có các góc trong a1, a2, ..., an là S = a1 + a2 + ... + an − nπ + 2π. Lời giải Tổng các góc trong của đa giác phẳng n-cạnh là (n − 2) π. Do đó, tương tự đối với tam giác, diện tích của đa giác trắc địa là hiệu của tổng các góc trong của nó với tổng các góc trong của đa giác phẳng có cùng số cạnh. Ta chia đa giác trắc địa thành các tam giác trắc địa bằng cách thêm các đường chéo, ta được (n − 2) tam giác. Tổng diện tích các tam giác bằng diện 11 Hình 4. Tam giác trên bán cầu. Tạp chí Epsilon, Số 23, 06/2023 tích đa giác và tổng các góc của các tam giác bằng tổng các góc của đa giác. Hình 5. Một đa giác trên mặt cầu được phân chia thành các tam giác. Áp dụng định lí Harriot - Girard đối với (n−2) tam giác và tổng hợp ta được S = a1 + a2 + ... + an − (n − 2) π = a1 + a2 + ... + an − nπ + 2π. Hình dung đa giác như Hình 6. Đặt thước đo góc tại mỗi góc, thêm −π trên mỗi cạnh và thêm 2π giữa mỗi mặt. Diện tích đa giác bằng tổng các số trên hình. Biểu diễn hình ảnh này rất hữu ích trong việc hiểu chứng minh của Legendre về công thức Euler. Với một đa diện lồi có V đỉnh, E cạnh và F mặt. Gọi x là điểm bất kì bên trong. Như Hình 7, xây dựng một hình cầu tâm x bao quanh đa diện. Ta có thể chọn hình cầu có bán kính bằng 1. Sử dụng các tia phát ra từ x, ta chiếu đa diện trên mặt cầu. Tưởng tượng rằng đa diện là một mô hình khung dây và x là một bóng đèn. Hình chiếu là bóng của khung dây trên bề mặt của mặt cầu. Khi đó các mặt của đa diện trở thành các đa giác trắc địa. Ta tính diện tích mặt cầu bằng hai cách. Trước hết, ta sử dụng công thức tính diện tích nổi tiếng để tìm ra diện tích mặt cầu đơn vị là: S = 4π. Sau đó tính tổng diện tích các mặt đa giác trên mặt cầu. Theo định lí Harriot - Girard, diện tích của mỗi mặt n - biên bằng tổng các góc trong trừ đi nπ − 2π. Gán tất cả các góc, cạnh và mặt trên mặt cầu, đặt các thước đo ở mỗi góc, thêm −π trên cả hai biên của mỗi cạnh và thêm 2π ở giữa mỗi mặt, tạo ra một mặt cầu như Hình 8. Mặc dù, tổng các góc tại mỗi đỉnh của đa diện nhỏ hơn 2π nhưng khi chiếu trên mặt cầu thì tổng các góc bằng 2π. Vì có V đỉnh nên ta có tổng các góc bằng 2πV. Mỗi cạnh thêm −2π mà có E cạnh nên tổng ta có −2πE. Mỗi mặt thêm 2π mà có F mặt nên tổng ta có 2πF. Vậy ta có: 4π = 2πV − 2πE + 2πF f ⇒ 2 = V − E + F. 2.4. Chứng minh của Euler Hình 6. Đa giác cầu. Hình 7. Phép chiếu một hình đa diện lên một mặt cầu. Hình 8. Đa diện cầu. Euler Xem thêm chứng minh đã đề xuất chứng minh công thức X = 2 bằng cách loại bỏ các đỉnh của khối đa diện lồi, mỗi lần loại bỏ một đỉnh cho đến khi chỉ còn lại một hình 12 này ở [4], trang 68. Tạp chí Epsilon, Số 23, 06/2023 chóp tam giác gồm bốn đỉnh. Ta bắt đầu với một khối đa diện lồi có V đỉnh, E cạnh và F mặt. Đầu tiên ta loại bỏ một đỉnh từ khối đa diện sao cho khối đa diện còn lại có ít hơn một đỉnh. Cần xác đỉnh số mặt và số cạnh. Gọi O là đỉnh sẽ được loại bỏ và giả sử có n mặt (do đó có n cạnh) có chung đỉnh O. Ta thấy đỉnh O có thể được loại bỏ bằng cách cắt đi n−2 khối chóp có đỉnh O. Ví dụ, khối đa diện ở Hình 9 có đỉnh O là giao của 5 mặt. Do đó nó được loại bỏ bằng cách cắt đi 3 khối chóp. Hình 9. Loại bỏ đỉnh O bằng cách cắt đi các khối chóp. Ta phải xét ba trường hợp đặc biệt. Trường hợp 1 Giả sử n mặt có chung đỉnh O đều là hình tam giác. Bằng cách cắt bỏ O, ta đồng thời loại bỏ n mặt này, hệ quả là tạo ra n − 2 mặt tam giác mới. Giả sử các mặt này không đồng phẳng, ta có số mặt của khối đa diện mới là F − n + (n − 2) = F − 2. (F là số mặt ban đầu) Trong quá trình ta cũng loại bỏ n cạnh giao nhau tại O, nhưng ta thêm n − 3 cạnh nằm giữa n − 2 mặt tam giác mới. Do đó số cạnh của khối đa diện mới là (E là số cạnh ban đầu) E − n + (n + 3) = E − 3. Ví dụ trong Hình 9, ban đầu khối đa diện có 11 mặt và 20 cạnh. Sau khi loại bỏ đỉnh O ta được khối đa diện mới có 9 mặt và 17 cạnh. Hình 10. 13 Tạp chí Epsilon, Số 23, 06/2023 Trường hợp 2 Giả sử một trong số các mặt giao nhau tại O không phải là hình tam giác (ví dụ mặt tô đen trong Hình 10). Khi khối chóp tam giác chia mặt đó bị loại bỏ thì mặt đó không hoàn toàn bị biến mất. Hơn nữa một cạnh mới sẽ được thêm vào khi mặt đó bị cắt làm hai. Do đó số cạnh và số mặt của khối đa diện mới đều lớn hơn 1 so với ban đầu. Ví dụ trong Hình 10, khối đa diện ban đầu có 12 mặt và 23 cạnh. Sau khi loại bỏ đỉnh O ta được khối đa diện mới có 12 − 2 + 1 = 11 mặt và 23 − 3 + 1 = 21 cạnh. Tổng quát lại, nếu khối đa diện ban đầu có s mặt không là hình tam giác có chung đỉnh O thì sau khi loại bỏ đỉnh O, số mặt và số cạnh sẽ nhiều hơn s đơn vị so với ban đầu. Vì vậy số mặt mới là F − 2 + s. Và số cạnh mới là E − 3 + s. Trường hợp 3 Giả sử hai mặt tam giác mới nằm cạnh nhau và đồng phẳng (ví dụ mặt được tô đen trong Hình 11). Chúng sẽ không tạo ra hai mặt phân biệt trong khối đa diện mới, mà tạo ra một mặt hình tứ giác. Vì vậy sẽ có ít hơn 1 mặt so với ban đầu. Và do không có cạnh giao giữa hai mặt nên cũng sẽ có ít hơn 1 cạnh so với ban đầu. Ví dụ trong Hình 11, khối đa diện có 11 mặt và 20 cạnh. Sau khi loại bỏ đỉnh O, khối đa diện còn lại 11 − 2 − 1 = 8 mặt và 20 − 3 − 1 = 16 cạnh. Vậy nếu thực hiện t lần thì sẽ có ít hơn t mặt và t cạnh. Vậy khối đa diện còn lại có số mặt là F − 2 + s − t. Và số cạnh là E − 3 + s − t. Hình 11. Các công thức này đại diện cho số mặt và số cạnh của khối đa diện sau khi một đỉnh được loại bỏ. Nếu ta lấy số cạnh mới trừ đi số mặt mới ta được (E − 3 + s − t) − (F − 2 + s − t) = 1. Có thể nói, hiệu của cạnh và mặt giảm đi 1 đơn vị sau khi loại bỏ 1 đỉnh. Do 14 Tạp chí Epsilon, Số 23, 06/2023 đó nếu loại bỏ n đỉnh thì hiệu của cạnh và mặt là E − F − n. Ta có thể kết luận được chứng minh của Euler. Ban đầu ta có một khối đa diện lồi với V đỉnh, E cạnh và F mặt. Giả sử ta lần lượt loại bỏ từng đỉnh một, thì sau n lần sẽ còn lại 4 đỉnh. Khi đó V − n = 4 hay n = V − 4. Khối đa diện còn lại có 4 đỉnh được gọi là khối chóp tam giác, có hiệu của cạnh và mặt là 6 − 4 = 2 mà theo lập luận ở trên là E − F − n. Do đó ta có công thức E − F − n = 2. (2.4.1) Và n = V − 4. (2.4.2) Thay (2.4.2) vào (2.4.1) và chuyển vế ta được V − E + F = 2. 2.5. Phương pháp loại bỏ tam giác Cauchy Xem thêm chứng minh đã đưa ra ý tưởng thêm vào hoặc loại bỏ một số đỉnh của hình đa diện sao cho giá trị biểu thức V − E + F không đổi. Cuối cùng ta được một tam giác đơn có V − E + F = 2. Cho một hình đa diện lồi có V đỉnh, E cạnh và F mặt. Đầu tiên ta sẽ biến đổi hình đa diện thành một đồ thị phẳng (Hình 12). Ta loại bỏ một mặt của hình đa diện, rồi di chuyển vào mặt này tất cả các đỉnh khác mà không làm thay đổi số lượng của chúng, ta sẽ có một hình phẳng của đa giác chứa trong một đường viền. Hình 12. Chiếu hình đa diện lên mặt đáy. Xét đồ thị phẳng lồi tạo ra từ các cạnh của đa diện. Chia đồ thị thành các hình tam giác bằng cách thêm đường chéo vào tất cả các miền không phải là hình tam giác (đồ thị thứ nhất Hình 13). Mỗi lần một đường chéo được thêm vào 15 này ở [4], trang 114. Tạp chí Epsilon, Số 23, 06/2023 thì số cạnh và số miền đều tăng thêm 1 đơn vị. Số đỉnh vẫn giữ nguyên. Như vậy, biểu thức V − E + F không đổi so với đồ thị ban đầu. Hình 13. Thứ tự tam giác loại bỏ từ đồ thị tam giác. Sau khi đồ thị đã được tam giác hóa, ta phân rã nó bằng cách loại bỏ các hình tam giác từ bên ngoài, cho đến khi chỉ còn lại một tam giác duy nhất. Một hình tam giác ở bên ngoài đồ thị có thể có một hoặc hai cạnh bên ngoài. Trong trường hợp một, tam giác được loại bỏ bằng cách bỏ đi một cạnh và một miền (đồ thị thứ hai Hình 13). Trong trường hợp hai, tam giác được loại bỏ bằng cách bỏ đi hai cạnh, một đỉnh và một miền (đồ thị thứ ba Hình 13). Trong cả hai trường hợp thì biểu thức V − E + F không đổi. Đến cuối cùng, ta được một đồ thị tạo bởi một tam giác đơn, có 3 đỉnh, 3 cạnh và 2 miền. Khi đó V − E + F = 3 − 3 + 2 = 2. Vậy V − E + F = 2 luôn đúng đối với đồ thị ban đầu. 3. Một số ứng dụng và bài toán liên quan Công thức đặc trưng Euler được sử dụng làm chìa khóa để chứng minh một số định lí và bài toán liên quan. Ta xét một số ví dụ sau. 3.1. Khối đa diện Platon Trong toán học, các khối đa diện Platon là các đa diện lồi đều. Chỉ có đúng 5 đa diện Platon đó là tứ diện đều (tetrahedron), hình lập phương (hexahedron), bát diện đều (octahedron), thập nhị diện đều (dodecahedron) và nhị thập diện đều (icosahedron). Các đa diện đều Platon được biết đến từ rất sớm trong thời kì cổ đại. Những đa diện đều Platon đầu tiên được tạo ra từ cách đây hơn 4000 năm và chúng được chạm khắc trên những khối đá. Xuất hiện từ rất sớm nhưng cho tới thời điểm cách đây hơn 2500 năm thì các quy luật toán học xung quanh vấn đề các khối đa diện đều Platon mới lần đầu tiên được đề cập tới và được nghiên cứu sâu rộng. Và cho tới khi nhà triết học, nhà thiên văn học và cũng là nhà hình học 16 Tạp chí Epsilon, Số 23, 06/2023 nổi tiếng Hy Lạp - Platon (khoảng 427-347 TCN) chỉ ra rằng chỉ có 5 khối đa diện đều thì chúng được biết gọi là các khối Platon. Các khối Platon gồm 5 đa diện đều tetrahedron, hexahedron, octahedron, dodecahedron và icosahedron (Hình 1). Dưới đây trình bày chứng minh tồn tại duy nhất 5 đa diện đều Platon nhờ đặc trưng Euler. Định nghĩa 1 Khối đa diện (H) được gọi là khối đa diện lồi nếu đoạn thẳng nối hai điểm bất kì của (H) luôn thuộc (H). Khối đa diện lồi đều là khối đa diện lồi có tính chất sau: a) Mỗi mặt của nó là một đa giác đều p cạnh. b) Mỗi đỉnh của nó là đỉnh chung của đúng q mặt. Khối đa diện đều như vậy được gọi là khối đa diện đều loại (p, q). Lời giải (Chỉ tồn tại duy nhất 5 đa diện đều Platon) Ta có đặc trưng Euler: V − E + F = 2. (3.1.1) Đầu tiên ta có mối liên hệ: pF = 2E = qV. Thật vậy, ta có p là số cạnh của mỗi mặt đa diện, F số mặt của khối đa diện, suy ra pF là tổng số cạnh của tất cả các mặt của khối đa diện. Mà một cạnh của đa diện kề với hai mặt của khối đa diện. Suy ra pF = 2E. Mặt khác ta lại có q là số mặt gặp nhau ở một đỉnh, V là tổng số đỉnh của khối đa diện. Suy ra qV v là tổng số đỉnh của tất cả các mặt của khối đa diện. Mặt khác, q là số cạnh gặp nhau ở một đỉnh. Mà mỗi cạnh liên kết với hai đỉnh của đa diện. Suy ra qV = 2E. Vậy: pF = 2E = qV . (3.1.2) Thế (3.1.2) vào (3.1.1) ta được: q− E +2Ep= 2. 2E ⇒1q+1p−12=1E. ⇒1q+1p=12+1E. (3.1.3) Bởi vì mỗi đa diện có ít nhất ba cạnh, khối đa diện có ít nhất ba mặt gặp nhau ở một đỉnh nên ta có p ≥ 3, q ≥ 3. Mặt khác nếu p, q cùng lớn hơn 3 thì sẽ dẫn đến p ≥ 4, q ≥ 4. Do đó p+1q≤14+14=12⇒12+1E≤12 1 17 Tạp chí Epsilon, Số 23, 06/2023 ⇒1E≤ 0. (3.1.4) Từ (3.1.4) suy ra điều vô lý. Do đó p, q không thể đồng thời lớn hơn 3 được. Suy ra p = 3 và q ≥ 3 hoặc p ≥ 3, và q = 3. Không mất tính tổng quát, giả sử p = 3. Thế vào (3.1.4) ta được: 1 q+13=12+1E. ⇒1q=16+1E>16. ⇒ 3 ≤ q < 6. Do q là số nguyên nên q chỉ có thể là 3, 4, 5. Từ đó suy ra e = 6, 12, 30. Một cách tương tự cho trường hợp q = 3. Ta cũng có p = 3, 4, 5. Vậy ta nhận được năm cặp số (3,3), (3,4), (3,5), (4,3), (5,3). Từ năm cặp số giá trị của (p, q) này cho ta năm đa giác đều cần chứng minh. 3.2. Trái bóng đá và bài toán phủ mặt cầu Trái bóng thường được phủ bởi các miếng da đen trắng, không biết có bao nhiêu người đã tò mò mà cầm lên đếm xem có bao nhiêu miếng da đen, bao nhiêu miếng da trắng. Cụ thể, mỗi mảnh ngũ giác đen gắn với 5 mảnh lục giác trắng khác. Mỗi mảnh lục giác gắn với 3 mảnh lục giác và 3 mảnh ngũ giác khác. Và thực tế, ta có thể chứng minh rằng chỉ có đúng một cách phủ kín bề mặt quả bóng theo kiểu như vậy, đó là phủ bởi 12 miếng da đen hình ngũ giác đều và 20 miếng da trắng hình lục giác đều. Để đơn giản, ta có thể coi quả bóng là một khối đa diện. Một đa diện lồi rất giống với trái bóng và nó sẽ biến thành trái bóng khi ta thực hiện một phép chiếu đa diện lồi trên lên một mặt cầu có tâm là tâm của đa diện đang xét. Một cách tưởng tượng, ta làm cong các mặt của đa diện từ các đa giác phẳng thành các đa giác cầu. Đa diện lồi được giới hạn bởi các mặt là các đa giác phẳng, các đa giác này được giới hạn bởi các đoạn thẳng, còn các đoạn thẳng được giới hạn bởi 2 đầu mút, chúng là các đỉnh của đa diện lồi. Gọi n là số mặt ngũ giác. Vì mỗi hình ngũ giác kề với 5 hình lục giác khác nên có 5n mặt lục giác. Con số này thực tế phải chia cho 3, vì mỗi mặt lục giác lại cùng lúc kề với 3 mặt ngũ giác. Khi đó số hình lục giác là 5n3. Mỗi đỉnh được tính ba lần (ứng với ba mặt), mỗi cạnh được tính hai lần (ứng với hai mặt) nên ta có: Số đỉnh: V =5n + 6.5n3 3= 5n; 18 Tạp chí Epsilon, Số 23, 06/2023 Số cạnh: E =5n + 6.5n3 2=15n2; Số mặt: F = n +5n3=8n3. Thay vào V − E + F = 2 ta được: n 6= 2 ⇒ n = 12. Như vậy cần phủ kín bề mặt quả bóng bằng đúng 12 mảnh ngũ giác và 20 mảnh lục giác. 4. Đặc trưng Euler và một số bài toán trong lý thuyết đồ thị Đặc trưng Euler có thể dùng để lập một số bất đẳng thức đối với các đồ thị phẳng và phát triển các định lí về lý thuyết đồ thị. 4.1. Bất đẳng thức đối với đồ thị phẳng Xem thêm ở [2], trang Hệ quả 1 Nếu G 586. là một đơn đồ thị phẳng liên thông có E cạnh, V đỉnh, với V ≥ 3. Khi đó E ≤ 3V − 6. Định nghĩa 2 Bậc của miền. Đó là số cạnh trên biên của miền đó. Khi một cạnh xuất hiện hai lần trên biên (tức là nó được vẽ hai lần khi vẽ biên) nó sẽ góp 2 đơn vị vào bậc của miền. Bổ đề 1 Tổng số bậc của các miền bằng đúng hai lần số cạnh của đồ thị Lời giải (Chứng minh Bổ đề 1) Trong đồ thị, mỗi cạnh là cạnh chung của hai miền. Do đó, tổng số bậc của miền bằng hai lần số cạnh của đồ thị. Lời giải (Chứng minh Hệ quả 1) Một đơn đồ thị phẳng liên thông khi vẽ trên một mặt phẳng sẽ chia mặt phẳng thành r miền. Bậc của mỗi miền ít nhất bằng 3 (vì ta chỉ xét đồ thị đơn nên không có cạnh bội để tạo ra miền bậc 2 và không có khuyên để tạo ra miền bậc 1). Đặc biệt, bậc của miền vô hạn ít nhất bằng 3 vì có ít nhất ba đỉnh trong đồ thị. 19 Theo Bổ đề 1 ta có 2E =P R Tạp chí Epsilon, Số 23, 06/2023 deg(R). Vì mỗi miền có bậc lớn hơn hoặc bằng 3 nên P R deg(R) ≥ 3r. Suy ra 2E3 ≥ r. Theo đặc trưng Euler ta có r = E − V + 2. Do đó E − V + 2 ≤2E3 ⇒ E ≤ 3V − 6. Ví dụ Chứng minh rằng đồ thị đầy đủ gồm 5 đỉnh K5 không là đồ thị phẳng. Lời giải Đồ thị K5 có năm đỉnh và mười cạnh. Vì bất đẳng thức E ≤ 3V − 6 không thỏa mãn đối với đồ thị này, do E = 10, 3V − 6 = 9. Vậy đồ thị K5 là không phẳng. Hệ quả 2 Nếu một đơn đồ thị phẳng liên thông có E cạnh, V đỉnh, với V ≥ 3 và không có chu trình độ dài 3 thì E ≤ 2V − 4. Lời giải (Chứng minh Hệ quả 2) Một đơn đồ thị phẳng liên thông khi vẽ trên một mặt phẳng sẽ chia mặt phẳng thành r miền. Theo Bổ đề ta có 2E =P R deg(R). Vì đồ thị không có khuyên hoặc cạnh bội và không có chu trình đơn độ dài 3 và bậc của miền vô hạn ít nhất là 4, nên mỗi miền có bậc ít nhất là 4. Do đó P R deg(R) ≥ 4r. Suy ra E2 ≥ r. Theo công thức Euler ta có r = E−V +2. Do đó E−V +2 ≤E2 ⇒ E ≤ 2V −4. Ví dụ Chứng minh đồ thị K3,3 là không phẳng. Lời giải Do đồ thị K3,3 không có chu trình độ dài 3 (Vì nó là đồ thị đầy đủ hai phía) nên ta có thể áp dụng Hệ quả 2. Đồ thị K3,3 có sáu đỉnh và chín cạnh. Vì E = 9 và 2V − 4 = 8, không thỏa mãn Hệ quả 2. Vậy đồ thị K3,3 là không phẳng. 20 Tạp chí Epsilon, Số 23, 06/2023 4.2. Định lý 4 Định lý 4 Nếu G là một đồ thị đơn phẳng có V ≥ 3 đỉnh thì G có nhiều nhất 3V − 6 Xem thêm ở [6], trang 77. cạnh. Lời giải Giả sử G có E cạnh và F miền. Có thể đếm số miền bởi số cạnh bao quanh miền, trong đó fk là số miền có k cạnh. Ta có F = f1 + f2 + f3 + f4 + ... Vì G là đồ thị đơn nên mỗi miền sẽ có ít nhất 3 cạnh. Do đó F = f3 + f4 + f5 + ... (4.2.1) Mà mỗi cạnh là cạnh chung của 2 miền nên 2E = 3f3 + 4f4 + 5f5 + ... (4.2.2) Từ (4.2.1) và (4.2.2) ta có: 2E − 3F = f4 + 2f5 + 3f6 + ... ≥ 0. (4.2.3) Sử dụng đặc trưng Euler V − E + F = 2 và (4.2.3) ta được: 3V − 6 = 3 (2 + E − F) − 6 = 3E − 3F ≥ E. Vậy G có nhiều nhất 3V − 6 cạnh. 4.3. Định lí 5 Định lý 5 Mọi đơn đồ thị phẳng có ít nhất một đỉnh có bậc nhỏ hơn hoặc bằng 5. Lời giải Theo chứng minh Định lí 4 ta có 2E − 3F = f4 + 2f5 + 3f6 + ... ≥ 0. (4.3.1) Giả sử, mỗi đỉnh có bậc nhỏ nhất là 6. Có thể đếm số đỉnh bởi số bậc của đỉnh, trong đó vilà số đỉnh có bậc i. V = v6 + v7 + v8 + v9 + ... 2E = 6v6 + 7v7 + 8v8 + 9v9 + ... 21 Do đó: Tạp chí Epsilon, Số 23, 06/2023 2E − 6V = v7 + 2v8 + 3v9 + ... ≥ 0. (4.3.2) Từ (4.3.1) và (4.3.2) suy ra 2E − 6V + 2 (2E − 3F) = 6E − 6V + 6F ≥ 0. Vì vậy V − E + F ≤ 0 (mâu thuẫn với công thức Euler). Do đó đồ thị phải có có ít nhất một đỉnh có bậc nhỏ hơn hoặc bằng 5. 5. Định lí Pick Định lý 6 (Định lý Pick (1899)) Diện tích của đa giác đơn P (không nhất thiết lồi) trên Xem thêm ở [3]. mặt phẳng với các đỉnh nguyên được cho bởi công thức SP = I +12B − 1, trong đó I là số điểm nguyên nằm trong P và B là số điểm nguyên nằm trên biên của P. Trước hết ta cần sử dụng kết quả bổ đề sau. Bổ đề 2 Mọi tam giác cơ bản đều có diện tích bằng 12. Định nghĩa 3 Tam giác cơ bản là tam giác có các đỉnh là các điểm nguyên, đồng thời trên biên và phần trong của nó không còn điểm nguyên nào khác. Lời giải (Chứng minh Bổ đề 2) Xét một tam giác cơ bản bất kì. Lấy một hình chữ nhật có các cạnh cùng phương với các trục tọa độ sao cho nó chứa cạnh tam giác và mỗi cạnh chứa ít nhất một đỉnh của tam giác. Vì hình chữ nhật có 4 cạnh và tam giác có 3 đỉnh nên có ít nhất một đỉnh của tam giác là đỉnh của hình chữ nhật, mà tam giác là cơ bản nên chỉ xảy ra tình huống ở Hình 14 H3 (bỏ qua các trường hợp tầm thường), tình huống mà một cạnh của tam giác là đường chéo của hình chữ nhật (trường hợp Hình 14 H1 có điểm nguyên L, Hình 14 H2 có điểm nguyên K.) 22 Tạp chí Epsilon, Số 23, 06/2023 Hình 14. Trong Hình 14 H3, tam giác cơ bản đang xét là OAB. Khi đó số điểm nguyên nằm trong tam giác OAB là nOAB = 0. Không mất tính tổng quát, ta giả sử M = (0; m) , P = (p; 0), B (b1; b2), ở đây m, p, b1, b2 là các số nguyên dương thỏa mãn p > b1 và m > b2. Số điểm nguyên nằm trong hình chữ nhật OMAP là nOMAP = (m−1)(p−1). Mà không có điểm nguyên nào trong số các điểm này nằm trên OA. Suy ra số điểm nguyên nằm trong tam giác OAP là nOAP =(m−1)(p−1) 2. Tính toán tương tự, ta có số điểm nguyên nằm trong tam giác OBR là nOBR =(b1−1)(b2−1) 2. Số điểm nguyên nằm trong tam giác BAS là nBAS =(p−b1−1)(m−b2−1) 2. Số điểm nguyên nằm trong hình chữ nhật BSP R là nBSP R = (b2 − 1) (p − b1 − 1). Số điểm nguyên nằm trên đoạn thẳng BS (khác B và S) là nBS = p−b1 −1. Số điểm nguyên nằm trên đoạn thẳng BR (khác B và R) là nBR = b2 − 1. Mặt khác, nOAP = 1 + nOAB + nOBR + nBAS + nBSP R + nBS + nBR. Suy ra (m−1)(p−1) 2 = 1 + (b1−1)(b2−1) 2 +(p−b1−1)(m−b2−1) 2 + (b2 − 1) (p − b1 − 1) + p − b1 − 1 + b2 − 1. Do đó mb1 − b2p = 1, bằng cách cộng trừ diện tích ta có SOAB =12. Lời giải Định lí Pick. Đầu tiên, ta chia đa giác P thành các tam giác cơ bản. Xét đồ thị G có đỉnh là các điểm nguyên nằm bên trong hay trên biên của P, các cạnh là cạnh của các tam giác cơ bản trong phép chia đang xét. Dễ thấy G là đồ thị phẳng, hữu hạn và liên thông và gồm f − 1 miền tam giác cơ bản. Gọi f là số mặt, eilà số cạnh trong (cạnh chung của hai tam giác cơ bản), eb là số cạnh biên (cạnh nằm trên cạnh của P) của G. 23 Vì diện tích của tam giác cơ bản bằng 12nên Tạp chí Epsilon, Số 23, 06/2023 SP =12(f − 1) (5.0.1) Do mỗi cạnh trong là cạnh chung của đúng hai tam giác cơ bản và mỗi cạnh biên là cạnh của đúng một tam giác cơ bản nên 3 (f − 1) = 2ei + eb. (5.0.2) Số đỉnh của G bằng I + B, số cạnh bằng ei + eb và số mặt bằng f nên theo công thức Euler ta có I + B − (ei + eb) + f = 2. (5.0.3) Ngoài ra, số cạnh ngoài và điểm ngoài là như nhau: B = eb . Nên từ (5.0.1), (5.0.2) và (5.0.3) ta có: SP = I +12B − 1. Vậy Định lí Pick được chứng minh. 6. Định lí Sylvester-Gallai Định lý 7 (Định lí Sylvester-Gallai) Trong mọi tập n > 2 điểm trên mặt phẳng, mà tất cả các điểm không nằm trên một đường thẳng, luôn tồn tại một đường thẳng chứa đúng hai điểm. Lời giải Nếu ta nhúng mặt phẳng R2 vào R3 gần mặt cầu S2.Khi đó với mỗi điểm trên mặt phẳng R2sẽ có một đường thẳng đi qua điểm đó và tâm hình cầu nên có sự tương ứng 1 − 1 giữa một điểm trên mặt phẳng với cặp điểm đối xuyên tâm trên mặt cầu. Một đường thẳng trên mặt phẳng tương ứng với một đường tròn lớn trên mặt cầu. Do đó ta có thể phát biểu lại Định lí như sau: Cho tập hợp n > 2 các cặp điểm đối xuyên tâm trên mặt cầu S2sao cho chúng không cùng nằm trên một đường tròn lớn. Khi đó có một đường tròn lớn chứa đúng một cặp điểm đối xuyên tâm. Giờ ta hoán đổi, thay thế mỗi cặp điểm đối xuyên tâm bởi một đường tròn lớn tương ứng trên mặt cầu. Nghĩa là, thay vì cặp điểm ±v ∈ S2ta xét đường tròn trực giao cho bởi Cv := x ∈ S2: ⟨x, v⟩ = 0 . (Nếu ta coi v là cực Bắc của hình cầu thì Cv là đường xích đạo). Vì vậy Định lí Sylvester-Gallai yêu cầu ta chứng minh: Cho tập hợp n > 2 các đường tròn lớn trên mặt cầu S2sao cho chúng không cùng đi qua một điểm. Khi đó luôn có một điểm là giao của đúng hai đường tròn lớn. Sự sắp xếp các đường tròn lớn tạo ra một đơn đồ thị phẳng trên S2có đỉnh 24 Xem thêm ở [6], trang 78. Tạp chí Epsilon, Số 23, 06/2023 là giao của các đường tròn lớn và chia chúng thành các cạnh. Tất cả các đỉnh đều có bậc chẵn và nhỏ nhất là bằng 4 (theo cách dựng). Do đó luôn tồn tại một đỉnh có bậc bằng 4. Do vậy, luôn có một điểm là giao của đúng hai đường tròn lớn. 7. Định lí về các đường thẳng đơn sắc Định lý 8 Cho một tập các điểm “trắng” và “đen” trên mặt phẳng, mà tất cả các điểm không nằm trên một đường thẳng. Khi ấy luôn tồn tại một đường thẳng “đơn sắc” (monochromatic line) chứa tối thiểu hai điểm cùng màu và không chứa điểm khác màu. Mệnh đề 1 Nếu S là một sự sắp xếp các đường thẳng trong mặt phẳng xạ ảnh với V đỉnh, E cạnh và F miền thì V − E + F = 1. Lời giải Trường hợp |S| = 2 ta có V = 1, E = 2, F = 2. Do đó V − E + F = 1. Giả sử công thức đúng với trường hợp |S| = n. Gọi l là đường thẳng không thuộc S và S′ = S ∪ {l} . Vì vậy S′ = n + 1. Gọi V′, E′, F′lần lượt là các đỉnh, cạnh và miền của S′. Bằng cách thêm l vào S bổ sung cho đỉnh, cạnh và miền được tạo ra. Đối với mỗi cạnh tạo ra bởi l trong E′, một miền của F được chia thành hai. Đối với mỗi đỉnh tạo ra bởi l trong V′, một cạnh của E cũng được chia thành hai. Do đó giá trị của biểu thức V − E + F không đổi. Vậy V − E + F = 1. Lời giải (Chứng minh Định lý) Ta sử dụng đặc trưng Euler cho đa giác (hay tổng quát hơn là cho đồ thị) trên bán cầu xạ ảnh đối ngẫu. Trong trường hợp này công thức đặc trưng Euler là V − E + F = 1 (theo mệnh đề) với V là số đỉnh, E là số cạnh và F là số miền. Giả sử không tồn tại đỉnh đơn sắc. Gọi rilà số các đa giác −i cạnh, c là số các góc tạo bởi hai màu khác nhau. Do không có đa giác − 2 cạnh nên Xem thêm ở [8]. F =X i≥3 ri. (7.0.1) Do mỗi cạnh là cạnh chung của đúng hai miền nên 2E =X i≥3 iri. (7.0.2) Do đa giác − 3 cạnh có thể có nhiều nhất 2 góc đơn sắc nên c ≤ 2r3 +X i≥4 iri. (7.0.3) 25 Tạp chí Epsilon, Số 23, 06/2023 Theo giả thiết mỗi đỉnh đều là đỉnh đơn sắc và mọi vòng cung khác màu giao nhau đều tạo ra ít nhất 4 góc. Do đó c ≥ 4V. (7.0.4) Từ (7.0.1), (7.0.2), (7.0.3) và đặc trưng Euler ta có V = 1 − F + E = 1 −X i≥3 ri +12X i≥3 iri = 1 −X i≥3 i 2− 1 ri ⇒ 4V = 4 + X i≥3 4 i 2− 1 ri = 4 + X i≥3 (2i − 1) ri = 4 + 2r3 + 4r4 + 6r5 + 8r6 + ... > 0 + 2r3 + 4r4 + 6r5 + 8r6 + ... ≥ c. Do đó c < 4V. Mâu thuẫn với điều (7.0.4). Suy ra điều cần chứng minh. Kết luận Chương trình và nội dung mới của môn toán trong trường phổ thông theo định hướng phát triển năng lực của học sinh. Một số kiến thức mới của toán học như hình học tổ hợp, hình học rời rạc, lí thuyết đồ thị, v.v. sẽ được cập nhật, cho phù hợp với cuộc sống (toán học trong công nghệ thông tin,...). Hy vọng Đẳng thức Euler, một công thức mang tính bản chất, sẽ được các giáo viên và học sinh quan tâm. Tài liệu [1] Stewart Ian (2015), 17 phương trình thay đổi thế giới, NXB Trẻ (Dịch từ: Stewart Ian (1945), 17 equations that changed the world) [2] Kenneth H.Rosen (1998), Toán học rời rạc ứng dụng trong tin học, NXB Khoa học và Kỹ thuật Hà Nội (Dịch từ: Kenneth H.Rosen, Discrete Mathematics and Its Applications- McGraw-Hill, 1994). [3] Nguyễn Trung Tuân, (Tháng 3-2018), Định lí Pick, https://nttuan.org/2017/03/18/topic-872/. [4] David S. Richeson (2012), Euler’s Gem: The Polyhedral Formula and the Birth of Topology, Oxford. [5] N. L. Biggs, E. K. Lloyd, R. J. Wilson (2009), Graph Theory 1736–1936, Oxford. [6] A. Martin, Z. Guter (2014), Proofs from the book, Springer-Verlag, Berlin, New York. [7] David Eppstein, "Twenty Proofs of Euler’s Formula: V − E + F = 2,"https://www.ics.uci.edu/ eppstein/junkyard/euler/. [8] Jonathan Lorand (2012), The Sylvester − Gallai Theorem, the Monochrome Line Theorem and Generalizations, Report for a Seminar on the Sylvester − Gallai Theorem. 26 Tạp chí Epsilon, Số 23, 06/2023 Có hay không lớp văn hóa Ấn trước lớp văn hóa Hán? Nguyễn Lê Anh Giới thiệu. Đây là số thứ 11 liên tục chúng tôi luôn đăng chuyên mục phám phá lịch sử thông qua tư duy lô-gích và phương pháp suy luận khoa học. Chúng tôi muốn giới thiệu với độc giả chúng ta có thể làm được những gì với tư duy toán học, thông qua những nghiên cứu miệt mài của nhà toán học Nguyễn Lê Anh. Trong số này, chúng tôi tiếp tục đăng những ghi chép của ông, những nổ lực không ngừng nghỉ cùng với những khám phá mới. Đây cũng là lời kêu gọi cộng đồng trong việc cùng chung tay xây dựng một tài liệu lịch sử cho dân tộc chúng ta - dân tộc Việt Nam. 27 Tạp chí Epsilon, Số 23, 06/2023 Vào thời kỳ trước Công Nguyên, sông Cà Lồ là một con sông rất lớn rộng tới 2 dặm, 700m. Nó là con sông chính chảy qua khu vực Bình Lệ Nguyên, là vùng có kinh tế phát triển nhất ở vùng đồng bằng Bắc Bộ. Con sông lớn thì luôn có tên. Tên của sông phải là thứ mọi người dân đều cảm nhận được nét đặc trưng để họ truyền khẩu. Hầu hết tên núi và tên sông xuất phát từ gốc chữ Hán đều liên quan tới các từ có nghĩa. Tên sông Cà Lồ không thể bắt nguồn từ âm Hán hay âm Việt. Trong từ điển tiếng Champa thì âm calaoh có nghĩa là "rắn nước". Cũng có thể cái tên Cà Lồ hàm ý ngoằn nghèo, mà cũng có thể con sông này có rất nhiều rắn nước. 泠Mê Linh huyện Tự mê là nhại thanh được tạo ra từ 米鹿 Độ thông dụng trong Hán ngữ cổ: rất thấp Độ thông dụng trong tiếng Trung hiện đại: rất thấp Bộ mễ 米 Bộ lộc 鹿là con hươu. Con này thoát ẩn thoắt hiện. Tự linh 泠là từ nhại thanh được tạo ra từ bộ thuỷ 1 và bộ linh 令 Độ thông dụng 泠trong Hán ngữ cổ: thấp Độ thông dụng 泠trong tiếng Trung hiện đại: thấp Bộ Thủy2là thu gon của bộ thủy 水. Bộ linh 令 Độ thông dụng trong Hán ngữ cổ: rất cao Độ thông dụng trong tiếng Trung hiện đại: rất cao nó hàm ý nói về mệnh lệnh, sự linh thiêng. Như vậy 泠là nhại thanh, tức sử dụng một số tự có âm gần đúng với âm thanh của người dân bản xứ mà nhại. Như vậy Mê Linh 泠trong các cổ thư Hán là từ nhại thanh của âm có sẵn. Âm này cũng không phải là âm Việt. Sông Hát, Cổ Loa cũng là những âm như vậy, không phải bắt nguồn từ âm Hán cũng như âm Việt. Như thế tên địa danh và các dòng sông thuộc khu vực liên quan tới Bà Trưng là thuộc một phân tầng lịch sử của dân tộc Việt. Vào khoảng 1200 năm trước Công Nguyên, có một dòng dân di cư khoảng từ 200 cho tới 2000 người dọc theo sông Thao tới vùng Phùng Nguyên. Họ được gọi là các Vua Hùng. Di chỉ khảo cổ cho thấy xuất hiện 4 chiếc Nha Chương là đặc trưng của nhóm người họ Phùng di cư từ vùng Tân Cương tới. Hậu duệ của dòng dân di cư này là cộng đồng họ Phùng ở vùng từ chân núi Ba Vì, qua Đường Lam và thành cổ Sơn Tây tới ven bờ sông Hồng nay. Các khảo cứu từ Trung Quốc cho thấy cộng đồng cư dân họ Phùng đã chuyển sang chế độ Phụ hệ từ rất lâu trước đó. Chế độ Phụ hệ quản lý xã hội tốt hơn là Mẫu hệ. Câu truyện Phù Đổng Thiên Vương có thể là dấu vết của các cuộc chiến bành trướng của hậu duệ các vua Hùng về phía nam. Khả năng rất lớn là sau 1200 năm thì khu vực Hatmon, Melinh và toàn bộ vùng Bình Lệ Nguyên (Vĩnh Phúc, Bắc Ninh) là các khu vực liền với Sơn Tây cũng đã là bước vào chế độ 1Phần bên trái của tự linh, do hạn chế, Epsilon không thể hiện được bộ này. 2xem chú thích trước. 28 Tạp chí Epsilon, Số 23, 06/2023 Phụ hệ. Cuộc chiến Bà Trưng đánh và tiêu diệt quá nửa trong số 20 nghìn quân của Mã Viện cho thấy, đây là một cuộc chiến rất lớn và khốc liệt. Cổ sử nhà Hán ghi rõ cuộc chiến này do đích thân Hán Quang Vũ Đế ra lệnh đánh. Như vậy 20 nghìn quân tham chiến của Mã Viện là quân thiện chiến. Vậy thì quân của Bà Trưng cũng phải ở mức độ thiện chiến nhất định mới có thể tiêu diệt được quá nửa quân của Mã Viện. Để có được quân chống lại Mã Viện, Bà Trưng phải huy động quân ở một khu vực tương đối rộng. Điều này cho thấy Bà Trưng là một nhà nước và khả năng rất cao đó là chế độ Phụ hệ. Các công trình khảo cổ đã chỉ ra thành Cổ Loa được đắp vào khoảng thời gian 260 trước Công Nguyên. Thủy Kinh Chú có ghi thành Nê Lê ven sông Trường Giang (đoạn từ sông Hồng tới sông Đuống) do A dục Vương, tức vua Ashoka, xây. Ở Bắc Bộ tất cả các thành đều được xây vào khoảng thế kỷ thứ 2 sau Công Nguyên, chỉ duy nhất thành Cổ Loa là được xây vào khoảng thế kỷ thứ 2 trước Công Nguyên. Như thế Thành Cổ Loa là thành Nê Lê do A Dục Vương Xây. Nội dung nghiên cứu "Khảo sát khảo cổ học di tích Khao Làng, xã Nghi Thạch, huyện Nghi Lộc, tỉnh Nghệ An"của nhà khảo cổ học Phạm Huy Thông, được công bố trên Tạp chí Khảo cổ học số 4 năm 1976 cho thấy khu vực Nghệ An có nhiều mỏ đồng tiền sử từ 3000 năm về trước. "Nghiên cứu địa chất khoáng sản các mỏ đồng xã Nghi Phú, huyện Nghi Lộc, tỉnh Nghệ An"của các tác giả: Nguyễn Văn Hùng, Vũ Quang Vinh và Đặng Văn Hùng. Bài báo này trình bày kết quả nghiên cứu địa chất khoáng sản của các mỏ đồng tại xã Nghi Phú, huyện Nghi Lộc, tỉnh Nghệ An, trong đó có đề cập đến niên đại của các mỏ đồng tiền sử. Như vậy khu vực Nghệ An có nhiều mỏ đồng và thiếc tiền sử 3000 năm tuổi. Vào thời kỳ 2800 năm cho tới 2000 về trước đồng chính là vàng. Đồng vừa được dùng làm tiền trong giao thương và tích lũy vốn, vừa được dùng để chế tác vũ khí và công cụ lao động. Vương quốc của Ashoka bao gồm toàn bộ Ấn Độ nay, qua Iran tới bờ Địa Trung Hải, lên quá vành đai Thảo Nguyên và tới tận phía Đông Dương. Một vương quốc rộng lớn như vậy với dân số ước tính vài chục triệu thì nhu cầu đồng là rất lớn. Cuộc chiến Ashoka phát động đánh về phía Đông là vào thời kỳ "cơn sốt Vàng (đồng)". Di tích khảo cổ và di tích được ghi nhận trong Thủy Kinh Chú cho thấy Ashoka đã tới Cổ Loa. Có lẽ A Dục Vương đã để lại dấu ấn là sự bắt đầu giao thương biển cùng với việc xuất hiện các đồ đồng "Đông Sơn"được đúc tinh xảo. Rất có thể vùng đất Cổ Loa - Mê Linh là nơi chịu ảnh hưởng của A Dục Vương. Vào cùng thời gian này, ở miền Trung Việt Nam xuất hiện vương quốc Champa. Rất có thể cuộc chiến Ashka đã kích thích các cuộc di dân. Theo dõi kỹ lịch sử Champa thì chúng ta có thể nhận ra đệm của tên gọi của các vị vua Champa là Bà. Bà Tấm - Po Nraop (chữ Hán: 婆) là em trai của vua Po Rome, và là vua của tiểu quốc Panduranga (Chiêm Thành) từ năm 1651 đến năm 1653. Bà Tranh - Po Saot hay Bà Tranh (Wan Dam, chữ Hán: 婆) là con trai của Po Rome, làm vua Chiêm Thành từ khoảng năm 1659 đến năm 1693. Po Rome là vua 29 Tạp chí Epsilon, Số 23, 06/2023 của tiểu quốc Panduranga (Chiêm Thành) từ năm 1627 đến năm 1651. Ông là con rể của Po Klong M’hnai. Năm 1631, Công nữ Ngọc Khoa được gả cho Po Rome. Dưới tiền triều Po Ehklang, Po Klong M’hnai được ban tước hiệu Maha Taha Theo từ điển Champa - Việt thì từ po Oπ⁄ [Bkt.] 1 d. ngài, trời, đấng. Có bạn nào biết tốt lịch sử Champa và các dòng họ Champa thì cho ý kiến. Rất có thể âm Bà trong tên gọi Bà Trưng là Po - 婆. Nếu vậy Bà Trưng là một vị vua đàn ông! Bà Trưng tức vua Trưng. Câu truyện "Chọi Trâu"chỉ có duy nhất ở cư dân Minangkabau (một dân tộc ở tây Sumatra, indonesia) và dân tộc Kinh. Các di chỉ văn hóa của Mianangkabau tương đồng hoàn toàn với các hoa văn trên trống đồng Ngọc Lũ. Truyền thuyết cũng như nét tương đồng văn hóa cho thấy Mianangkabau có sự tương đồng văn hóa với người Kinh ở đồng bằng Bắc Bộ. Các tính toán cho thấy đã có khoảng vài trăm người di cư tới vùng tây Sumatra vào khoảng trước Công Nguyên 200 năm. Kết hợp với truyền thuyết An Dương Vương (A Dục Vương) đánh nhau với Triệu Đà, thu chạy ra biển ở của Hiền. Rõ ràng là truyền thuyết có nhắc tới việc Mị Châu mặc áo lông, tức mùa đông. Điều này cũng phù hợp với sự kiện quân Triệu Đà muốn di chuyển từ phía bắc tới Cổ Loa bằng đường biển thì phải là lúc có gió mùa Đông Bắc. Ngoài ra chỉ có thể di chuyển tới tây Sumatra vào mùa đông khi có các đợt gió bắc thổi về phía nam. Như vậy khả năng rất cao Triệu Đà đã có một lượng nhỏ quân, lợi dụng gió mùa Đông Bắc, di chuyển theo ven biển tiến đánh A Dục Vương. A Dục Vương thua trận và đã chạy từ cửa Hiền, và sau đó lên tàu chạy tới vùng tây Sumatra của Indonesia. Đây là khu vực gần với Ấn Độ. Khu vực cửa Hiền nằm cách không xa các mỏ đồng tiền sử ở Nghệ An. Nhiều trống đồng Đông Sơn được xác định là thuộc định dạng và có hoa văn như trông đồng Ngọc Lũ được tìm thấy ở Malaysia. Vậy khả năng cao nơi đây từng là cảng biển, có nhiều tàu thuyền viễn dương. Trên thực tế thì không có một tài liệu nào vào thời kỳ đầu Công Nguyên khẳng định Ashoka (A Dục Vương) xuất hiện ở đâu, ngoài sự kiện thành Nê Lê ở trong Thủy kinh Chú. Kết hợp các sự kiện lại chúng ta có thể nhận thấy có một phân tầng lịch sử chịu sự ảnh hưởng của văn hóa Ấn Độ trước khi chịu sự ảnh hưởng của văn hóa Hán. Theo đó, khu vực Đông Dương là nơi có nhiều mỏ đồng lộ thiên. Vì thế vào khoảng năm 260 trước Công Nguyên, do các cơn sốt đồng mà cư dân nhiều nơi đã di cư về khu vực Đông Dương. Văn minh Funan là cùng thời với nền văn hóa Óc Eo, khoảng từ thế kỷ thứ 1 tới thứ 7. Chúng ta có thể phải mất rất nhiều thời gian để có thể tìm hiểu được mối liên quan giữa dân tộc Kinh với Champa với Funan và Chân Lạp. 30 Tạp chí Epsilon, Số 23, 06/2023 Tiếp cận tổng thể tới dạy Ngôn Ngữ và Toán Nguyễn Ái Việt Giới thiệu. Nếu như Trí khôn nhân tạo sẽ thay đổi yêu cầu về nhân lực trong tương lai, vai trò của các nhà bác học đa ngành (polymaths) và các chuyên gia tổng quát (expert generalists) sẽ ngày càng lớn, với vai trò tạo ra kết nối giữa các mảng tri thức chuyên ngành, tạo ra giá trị mới. 31 Tạp chí Epsilon, Số 23, 06/2023 1. Nếu như Trí khôn nhân tạo sẽ thay đổi yêu cầu về nhân lực trong tương lai, vai trò của các nhà bác học đa ngành (polymaths) và các chuyên gia tổng quát (expert generalists) sẽ ngày càng lớn, với vai trò tạo ra kết nối giữa các mảng tri thức chuyên ngành, tạo ra giá trị mới. Điều đó có nghĩa là Toán học và Ngôn ngữ sẽ trở lại vai trò như chúng đã từng có ở Kỷ nguyên Phục hưng và Bừng sáng. Tuy nhiên, trong tương lai Toán học và Ngôn Ngữ sẽ phải có một diện mạo mới. Vì vậy việc dạy Toán và Ngôn Ngữ sẽ phải thay đổi đến tận gốc rễ từ Tiểu học hoặc sớm hơn nữa. Trong bài này, chúng tôi sẽ trình bày quan điểm dạy Toán và Ngôn Ngữ theo quan điểm tổng thể. Thay vì kêu gọi một chương trình cải cách giáo dục, mà chắc chắn sẽ biến dạng và chẳng đi đến đâu, chúng tôi dự định tiến hành tiếp cận này bằng các khóa bổ trợ, nhằm vá cách mảnh trí thức manh mún, có nguy cơ trở thành vô dụng. 2. Vào những năm 1990, người ta đã nhận ra rằng việc dạy Ngôn Ngữ theo cách phân tích thành các đơn vị nhỏ và tiến hành lắp ráp lại từ đầu ngày càng trở nên lạc hậu, tốn công sức, thời gian và hiệu quả thấp. Cách tiệm cận tổng thể tới Ngôn ngữ (Whole language approach) đã được chấp nhận rộng rãi [1]. Theo đó, việc dạy Ngôn ngữ được tiến hành theo các dạng thức mà người học sẽ tìm thấy nó trong cuộc sống. Việc giản lược Ngôn ngữ thành các câu mẫu ngô nghê ít gặp trong cuộc sống sẽ đòi hỏi nhiều công sức soạn giáo trình, và ngược lại cũng đòi hỏi nhiều công sức hơn để học viên xóa chúng khỏi bộ nhớ để thích nghi với Ngôn ngữ được sử dụng thực sự trong cuộc sống. Các thử nghiệm thành công đã chứng tỏ cách tiệm cận tổng thể có nhiều ưu thế trong đào tạo hiệu quả hơn. Trong lớp học, thay vì "xay"và "khoan"các mảnh vụn ngôn ngữ, để chết chìm trong những đống quy luật vô nghĩa, người học được tiếp cận tới bức tranh tổng thể, và nhận biết các vấn đề về sử dụng ngôn ngữ để tiếp thụ và diễn đạt các ý tưởng phức tạp. 3. Cách tiếp cận này tới Ngôn Ngữ lập tức có ảnh hưởng tới chương trình học. Thay vì phải đắm mình vào các vấn đề "tự thân"của Ngôn Ngữ, do băm nát Ngôn ngữ thành cách mảnh vụn, rồi đi tìm các quy tắc, mà tuyệt đại đa số có ngoại lệ, để ghép chúng lại với nhau, môn Ngữ Văn được quan niệm là công cụ để nhận thức. Giờ Ngữ Văn hoặc học Tiếng, sẽ dùng để đọc, viết, nghe và nói về các đề tài xuyên suốt qua các mảng tri thức khác nhau. Ngữ Văn sẽ hòa trộn với tri thức và tư duy thay vì bị tách biệt thành quy tắc và tín điều phải chấp nhận. Trong cách tiếp cận này, học sinh sẽ thực sự thao tác, xây dựng ngữ nghĩa trong các project do họ tiến hành, mà kết quả không phải là duy nhất hay phải chấp nhận thụ động từ giáo viên. 4. Vấn đề đặt ra là có thể xây dựng một quan điểm tương tự đối với việc dạy Toán hay không [2]. Cá nhân tôi đã có những thử nghiệm để chứng minh quan điểm: Học sinh kém và ghét Toán bắt đầu từ yếu kém trong việc sử dụng ngôn ngữ. Trước khi giúp các học sinh này học Toán cần phải sửa lỗi sử dụng Ngôn Ngữ trong Toán học và điều đó gắn liền với việc dạy sử dụng Logic trong Ngôn Ngữ. Do đó, tôi đi đến một quan điểm dạy Ngôn Ngữ kết hợp với dạy Toán sẽ giúp phát triển tư duy của trẻ một cách lành mạnh và hiệu quả cho cả học Ngôn Ngữ và học Toán. Nếu có thể dạy Ngôn Ngữ theo tiệm cận tổng thể, quan điểm này cũng sẽ dẫn tới dạy toán theo tiệm cận tổng thể. Chúng ta sẽ coi đó là một giả thuyết và sẽ tìm cơ sở cho nó. 32 Tạp chí Epsilon, Số 23, 06/2023 5. Trước hết, chúng ta sẽ xem xét sự tương đồng của Toán học và Ngôn Ngữ. Việc đưa ra một định nghĩa chính xác cho Toán học khó khăn là bởi vì có 3 cách tiếp cận đến Toán học: i. Các tiếp cận logic (coi logic là cơ sở của Toán học), ii. Tiếp cận tạo dựng (coi số tự nhiên là cơ sở của Toán học) và iii. Tiếp cận hình thức (xây dựng Toán học dưới dạng một chuỗi suy diễn hình thức, phi mâu thuẫn). Tuy định lý Godel đã phủ nhận quan điểm iii., nhưng tiếp cận hình thức vẫn là cách tiếp cận chủ đạo hiện nay từ các trường và viện nghiên cứu, tới Tiểu học, thậm chí Mẫu giáo. Do đó gần đây, đã có nhiều ý kiến quanh việc khắc phục các hệ quả của quan niệm này trong giáo dục [2]. 6. Tôi cho rằng, ít nhất đối với đa số người ứng dụng Toán trong cuộc sống, Toán học có thể được coi là một ngôn ngữ để mô tả (định tính và định lượng), kiểm tra tính đúng đắn, suy diễn các tri thức khác nhau được trừu tượng hóa. Như vậy, dạy Toán ở Phổ thông, đặc biệt ở Tiểu học nên bắt đầu bằng các hoạt động phổ quát như đếm, đo, xác định vị trí, thiết kế, quan sát, suy diễn và giải thích [3]. Đương nhiên các hoạt động này phải được thực hiện trong các văn cảnh cụ thể của các vấn đề khoa học, công nghệ, xã hội, kinh tế,... Có người cho rằng Toán học khác Ngôn Ngữ ở chỗ nó có những nội dung độc lập với các đối tượng mà nó mô tả. Tuy nhiên Ngôn Ngữ cũng có các nội dung tự thân của nó. Tôi đồng ý với quan điểm cho rằng Toán học và Ngôn ngữ là hai thiết chế xã hội cơ bản có liên quan với nhau. 7. Trong tiếp cận Toán học tổng thể (Whole Mathematics) sẽ có các đặc trưng [2] a. Đào tạo Toán học phải tiếp cận tới Toán học như một tổng thể, tìm kiếm tối đa điểm liên quan giữa các lĩnh vực Toán học khác nhau b. Nhấn mạnh tới các chức năng của Toán học c. Dạy theo các dự án, có mục tiêu xác định, để lựa chọn công cụ Toán học phù hợp d. Học sinh sẽ đóng vai trò chủ động hơn trong việc xác định nội dung học (thông qua các chuyên đề tự chọn, tự nghiên cứu) e. Toán cần được tích hợp theo các nội dung ứng dụng f. Các hình thức mô tả toán học khác nhau của cùng một khái niệm cần được khai thác g. Ý nghĩa cần được đặt ưu tiên cao hơn kỹ năng biến đổi công thức h. Toán không phải là kỹ năng tính, mà cần thể hiện các ý tưởng mà nó biểu diễn một cách sâu sắc. 8. Hơn nữa, theo quan điểm của tôi, Toán học có thể dạy cùng với Ngôn Ngữ, dựa trên logic, để diễn tả những thông điệp khác nhau về Văn hóa, Xã hội, Khoa học và thế giới xung quanh ta. Như vậy, Toán học sẽ đi vào thực tế mạnh mẽ như Ngôn Ngữ, giúp Ngôn Ngữ thêm phần tư duy, tránh sáo mòn áp đặt. Đó chính là cái chúng ta cần trong tương lai của Trí khôn nhân tạo và Tính toán phỏng não. Đó là thời đại mà các tư tưởng Phục Hưng và Bừng sáng sẽ trở lại đưa nền văn minh này khỏi các bế tắc hiện nay. 33 Tạp chí Epsilon, Số 23, 06/2023 [1] Davenport, M. R. & Watson, D. J. (1993) Whole language: philosophy and work-in progress, in S. K. Brady & T. M. Sills (Eds) Whole Language, History, Philosophy and Practice, pp. 1-17. Dubuque: Kendall/Hunt. [2] A.L.B.Diaz, A Whole Language Approach in Mathematics Classrooms, Cur riculum Studies, 6:1, (1998) 97-112, [3] Bishop, A. J. (1988) Mathematical Enculturation: a cultural perspective on mathematics education. Dordrecht: Kluwer Academic Publishers. 34 Tạp chí Epsilon, Số 23, 06/2023 Sự cần thiết của bất đối xứng Trần Văn Trản Giới thiệu. Các nhà vật lý lý thuyết say mê tính đối xứng, và nhiều người tin rằng các phương trình nên phản ánh vẻ đẹp này. Trong thực tế, phương trình toán học được xây dựng trên tính đối xứng đã dự đoán chính xác sự tồn tại của phản vật chất. Nhưng sẽ là sai lầm nguy hiểm khi đánh đồng sự thật và cái đẹp với sự đối xứng. Cả sinh vật sống lẫn bản thân Vũ trụ đều không đối xứng hoàn hảo. 35 Tạp chí Epsilon, Số 23, 06/2023 Hình thể con người có vẻ như là đối xứng. Trên thực tế Thượng Đế dùng một lớp da thịt để tạo nên sự đối xứng đó còn cấu trúc bên trong thì rất không đối xứn. Đối xứng là đẹp, nhưng bất đối xứng là lý do tại sao Vũ trụ và sự sống tồn tại. Vũ trụ có sự bất đối xứng, nhưng đó là một điều tốt. Sự không hoàn hảo là điều cần thiết cho sự tồn tại của các ngôi sao và thậm chí cả sự sống. Các nhà vật lý lý thuyết say mê tính đối xứng, và nhiều người tin rằng các phương trình nên phản ánh vẻ đẹp này. Trong thực tế, phương trình toán học được xây dựng trên tính đối xứng đã dự đoán chính xác sự tồn tại của phản vật chất. Nhưng sẽ là sai lầm nguy hiểm khi đánh đồng sự thật và cái đẹp với sự đối xứng. Cả sinh vật sống lẫn bản thân Vũ trụ đều không đối xứng hoàn hảo. Ta biết, những người thuận tay trái là thiểu số với tỷ lệ khoảng 1:10. Nhưng đừng nhầm lẫn: Vũ trụ yêu thích sự thuận tay trái, từ các hạt hạ nguyên tử cho đến chính sự sống. Trên thực tế, nếu không có sự bất đối xứng cơ bản này trong Tự nhiên, Vũ trụ sẽ là một nơi nhạt nhẽo, hầu hết chứa đầy bức xạ và không có các ngôi sao, hành tinh hay sự sống. Tuy nhiên, vẫn có một ý tưởng phổ biến trong các ngành khoa học vật lý thúc đẩy sự hoàn hảo về mặt toán học - được biểu thị bằng tính đối xứng - như bản thiết kế của Tự nhiên. Và, như thường lệ, chúng ta bị lạc vào một tình huống mập mờ như phải chọn phe: “tất cả là đối xứng” hay “chủ nghĩa không hoàn hảo”. Nhà vật lý vĩ đại Paul Dirac đã nói: “Điều quan trọng là phải có vẻ đẹp trong phương trình bất kỳ hơn là để chúng phù hợp với thí nghiệm.” Nếu bất kỳ nhà vật lý nào khác ít được biết đến hơn nói như vậy, họ có thể sẽ bị các đồng nghiệp chế giễu. Nhưng đó là Dirac, và phương trình tuyệt vời của ông, được xây dựng dựa trên các khái niệm đối xứng, đã dự đoán sự tồn tại của phản vật chất, thực tế là mọi hạt vật chất (như electron và quark) đều có một phản hạt đồng hành. Đó là một thành tựu thực sự đáng kinh ngạc - toán học đối xứng, được áp dụng cho một phương trình, đã hướng dẫn con người khám phá toàn bộ lĩnh vực phản vật chất tồn tại song song. Chẳng trách Dirac lại sùng bái thần Đối Xứng đến vậy. Nó hướng dẫn suy nghĩ của ông hướng tới một khám phá đáng kinh ngạc Lưu ý rằng phản vật chất không có ý nghĩa lập dị như người ta tưởng. Các phản hạt không bay lên trong trường hấp dẫn. Chúng có một số tính chất vật lý bị đảo ngược, đáng chú ý nhất là điện tích. Vì vậy, phản hạt của electron tích điện âm, được gọi là positron, có điện tích dương. Nhưng ở đây có vấn đề mà Dirac đã lãng quên. Các định luật Tự nhiên quy định hành vi của các hạt cơ bản dự đoán rằng vật chất và phản vật chất phải phong phú như nhau, nghĩa là chúng sẽ xuất hiện theo tỷ lệ 1:1. Mỗi electron có một positron tương ứng. Tuy nhiên, nếu sự đối xứng hoàn hảo này xảy ra thì một phần nhỏ của giây sau Vụ nổ lớn, vật chất và phản vật chất sẽ bị triệt tiêu thành bức xạ (chủ yếu là photon). Nhưng rõ ràng đó không phải là những gì đã xảy ra. Khoảng một trong một tỷ (đánh giá thô) hạt vật chất tồn tại dưới dạng dư thừa. Và điều đó thật tốt, bởi vì mọi thứ chúng ta thấy trong Vũ trụ - các thiên hà, các ngôi sao, các hành tinh và các vệ tinh của chúng, sự sống trên Trái đất, mọi loại vật chất kết khối, sống và không sống - đều đến từ phần 36 Tạp chí Epsilon, Số 23, 06/2023 dư thừa nhỏ bé này nhờ sự bất đối xứng này giữa vật chất và phản vật chất. Trái ngược với sự đối xứng và vẻ đẹp được mong đợi của vũ trụ, các nghiên cứu trong những thập kỷ qua đã chỉ ra rằng các quy luật Tự nhiên không áp dụng như nhau cho vật chất và phản vật chất. Cơ chế nào đã tạo ra sự dư thừa nhỏ bé này-sự không hoàn hảo chịu trách nhiệm cho sự tồn tại của chúng ta, là một trong những câu hỏi mở lớn nhất trong vật lý hạt và vũ trụ học. Theo ngôn ngữ của các đối xứng bên trong (“bên trong” nghĩa là thay đổi tính chất của một hạt) và đối xứng bên ngoài (“bên ngoài” giống như chuyển động quay của một vật thể), tồn tại một phép toán đối xứng bên trong làm thay đổi một hạt vật chất thành một phản vật chất. Hoạt động này được gọi là “liên hợp điện tích” và được biểu thị bằng chữ in hoa C. Sự bất đối xứng vật chất-phản vật chất quan sát được ngụ ý rằng Thiên nhiên không biểu thị tính đối xứng liên hợp điện tích: trong một số trường hợp, các hạt và phản hạt của chúng không thể biến thành nhau. Cụ thể, đối xứng C bị vi phạm trong các tương tác yếu, lực chịu trách nhiệm cho sự phân rã phóng xạ. Thủ phạm là neutrino, loại hạt kỳ lạ nhất trong tất cả các hạt đã biết do chúng đi xuyên qua vật chất mà hầu như không bị cản trở. Có khoảng một nghìn tỷ neutrino mỗi giây đến từ Mặt trời và đi qua bạn ngay bây giờ. Để biết tại sao neutrino vi phạm đối xứng C, chúng ta cần thêm một đối xứng bên trong gọi là tính chẵn lẻ, được biểu thị bằng chữ P. Một “phép tính chẵn lẻ” biến một vật thể thành hình ảnh phản chiếu của nó. Ví dụ, bạn không phải là bất biến chẵn lẻ. Hình ảnh phản chiếu của người có trái tim ở phía bên phải. Đối với các hạt, tính chẵn lẻ liên quan đến cách chúng quay như các đỉnh. Nhưng các hạt là những đối tượng lượng tử. Điều này có nghĩa là chúng chỉ có thể quay với số vòng được “lượng tử hóa”, nghĩa là chúng chỉ có thể quay theo một số cách, giống như các bản ghi đĩa hát kiểu cũ chỉ có thể phát ở ba tốc độ: 33, 45 và 78 vòng/phút. Lượng spin nhỏ nhất mà một hạt có thể có là một "tốc độ"quay. Nó giống như một cái đỉnh quay thẳng lên. Nhìn từ trên xuống, nó có thể quay theo chiều kim đồng hồ hoặc ngược chiều kim đồng hồ. Electron, quark và neutrino là như vậy. Chúng ta nói rằng chúng có vòng quay 1/2 và nó có thể là +1/2 hoặc -1/2, hai tùy chọn tương ứng với hai hướng quay. Áp dụng phép toán C trên một neutrino thuận tay trái, chúng ta sẽ thu được một phản neutrino thuận tay trái. Vấn đề là, không có phản neutrino thuận tay trái nào trong Tự nhiên. Chỉ có neutrino thuận tay trái. Các tương tác yếu, tương tác duy nhất mà neutrino cảm nhận được (ngoài lực hấp dẫn), vi phạm đối xứng liên hợp điện tích. Đó là sự không vui của những người yêu thích sự đối xứng. Nhưng hãy tiến thêm một bước nữa. Nếu chúng ta áp dụng cả C và P (tính chẵn lẻ) cho một neutrino thuận tay trái, chúng ta sẽ nhận được một phản neutrino thuận tay phải: C biến neutrino thành phản neutrino và P biến thuận tay trái thành thuận tay phải . Và thu được phản neutrino thuận tay phải! Dường như có một sự may mắn. Các tương tác yếu vi phạm C và P một cách riêng biệt nhưng dường như thỏa mãn hoạt động đối xứng CP kết hợp. Trong thực tế, điều này có nghĩa là các phản ứng liên quan đến các hạt thuận tay trái sẽ xảy ra với cùng tốc độ như các phản ứng liên quan đến các phản hạt thuận tay phải. Có hy vọng rằng Tự nhiên là đối xứng CP trong tất cả các tương 37 Tạp chí Epsilon, Số 23, 06/2023 tác đã biết. Người đẹp đã quay trở lại. Nhưng, năm 1964, James Cronin và Val Fitch phát hiện ra một sự vi phạm nhỏ đối xứng CP kết hợp trong sự phân rã của một hạt gọi là kaon trung hòa, được ký hiệu là K0. Về cơ bản, K0 và các phản hạt của chúng không phân rã với tốc độ như lý thuyết đối xứng CP dự đoán. Cộng đồng vật lý lại bị sốc. Vẻ đẹp đã lại biến mất. Và nó đã không bao giờ phục hồi. Vi phạm CP là một sự thật của Tự nhiên. Vi phạm CP thậm chí còn có một hàm ý sâu sắc và bí ẩn hơn: các hạt cũng chọn một hướng thời gian ưa thích. Sự bất đối xứng của thời gian, dấu hiệu của một Vũ trụ đang mở rộng, cũng xảy ra ở cấp độ vi mô! Đây là một chủ đề rất lớn. Còn một sự thật bùng nổ khác về sự không hoàn hảo. Cuộc sống cũng được “trao tay”: các axit amin và đường bên trong tất cả các sinh vật sống từ amip đến nho, cá sấu đến con người đều thuận tay trái và tay phải. Trong phòng thí nghiệm, người ta tạo ra hỗn hợp 50:50 của các phân tử thuận tay trái và thuận tay phải, nhưng đó không phải là những gì chúng ta thấy trong Tự nhiên. Cuộc sống hầu như chỉ ưu tiên axit amin thuận tay trái và đường thuận tay phải. Một lần nữa, đây là một câu hỏi khoa học mở rất lớn. 38 Tạp chí Epsilon, Số 23, 06/2023 Một vài tính chất hình học trên một cấu hình về Tỷ số Vàng Trần Quang Hùng Giới thiệu. Bài viết giới thiệu một tính chất hình học trên một cấu hình về phép dựng Tỷ số Vàng dùng hình vuông nội tiếp trong một tam giác vuông cân. 39 Tạp chí Epsilon, Số 23, 06/2023 1. Mở đầu Trong [1], tác giả đã giới thiệu một phép dựng Tỷ số Vàng sử dụng hình vuông nội tiếp tam giác vuông cân như sau Định lý 1 Cho tam giác vuông cân ABC cùng với đường tròn ngoại tiếp của nó, nội tiếp một hình vuông DEF G với cạnh F G dọc theo cạnh huyền AB. Nếu cạnh DE kéo dài cắt đường tròn ngoại tiếp tại P, thì E chia DP theo Tỷ số √5+1 Vàng φ = 2(xem Hình 1). C D E P B A G F Hình 1. Phép dựng Tỷ số Vàng với hình vuông nội tiếp tam giác vuông cân. Phép dựng hình vuông DEF G nội tiếp tam giác vuông cân ABC có thể được mô tả đơn giản qua hình vẽ dưới đây (xem Hình 2). C D E Q P B O A G F Hình 2. Mô tả phép dựng hình vuông nội tiếp tam giác vuông cân. Trong việc tìm hiểu cấu hình về Tỷ số Vàng của Định lý 1, chúng tôi nhận thấy đây là một cấu hình thú vị, có thể chứa đựng nhiều bài toán mới. Thông qua một ví dụ là Định lý 2 dưới đây, chúng tôi muốn giới thiệu với bạn đọc cấu hình của Định lý 1. Chúng tôi đề xuất một tính chất hình học như sau 40 Tạp chí Epsilon, Số 23, 06/2023 Định lý 2 Cho tam giác ABC vuông cân tại A và nội tiếp trong đường tròn (O). Dựng hình vuông BCNM vào trong tam giác ABC. Các đường thẳng OM và ON cắt các đường thẳng AB và AC lần lượt tại F và E. Tia F E cắt (O) tại D. Khi đó đường thẳng nối trực tâm và điểm Lemoine của tam giác DMN đi qua A (xem Hình 3). M N H L A F ED B C O Hình 3. Minh họa cho Định lý 2. Chú ý. Định lý 2 cũng đã tác giả được giới thiệu như một bài toán trong [2]. 2. Một chứng minh cho Định lý 2 Trong mục này chúng tôi giới thiệu lời giải của tác giả Nguyễn Cung Thành từ [2] (xem Hình 4). Lời giải Theo Định lý 1 thì F E=1 + √5 F D 2. Sử dụng hệ tọa độ Đề-các vuông góc (xem [8]), ta cho các điểm có tọa độ như sau B(0, 0), C(2, 0), N(2, 2), M(0, 2), A(1, 1). Từ −→AF =13−−→AB = (−13, −13) ta thu được F 23,23 . Tương tự thì E 43,23 . 41 Tạp chí Epsilon, Số 23, 06/2023 Hình 4. Minh họa cho chứng minh Định lý 2. Do đó −−→F E 23, 0 . Kết hợp với Tỷ số Vàng từ Tỷ số Vàng F D F E =1+√5 2ta thu được −−→F D 1 + √5 ! và vì thế nên D 3, 0 3 + √5 3,23!. Mặt khác, không khó để tính được MN2 = 4. Đồng thời từ tọa độ của D, ta dễ tính được 3, DN2 =2(5 −√5) DM2 =2(5 + √5) 3, vì thế nên MN2 + DM2 + DN2 =323. Gọi L là điểm Lemoine của tam giác △DMN thì ta có đẳng thức vector (thu được từ tính chất tâm tỷ cự của điểm Lemoine [6]): MN2.−→LD + DN2·−−→LM + DM2·−−→LN =−→0 . Từ đây ta dễ tính được hoành độ của L là xL =MN2· xD + DM2· xN + DN2· xM MN2 + DM2 + DN2 = Và tương tự với tung độ của L là yL =32. 42 √5 + 4 4. Tạp chí Epsilon, Số 23, 06/2023 Ta thu được L √5 + 4 4,32!. Gọi H 3+√5 3, a là trực tâm của tam giác △DMN. Dễ thấy DH ⊥ BC nên hoành độ của H và của D bằng nhau. Vậy ta giả sử ! . Từ −−→DN = 3−√5 −−→MH = 3 + √5 3, a − 2 3,43 và DN ⊥ MH, ta thu được −−→DN ·−−→MH = 0 hay 49 +4(a−2) 3 = 0. Ta suy ra a =53. Như vậy ta có tọa độ các điểm √5 + 4 4,32!, H 3 + √5 A(1, 1), L 3,53!. Từ đó ta thấy phương trình tổng quát (xem [8]) của đường thẳng AL là x − √5 2y + √5 − 2 2= 0. Không khó để thấy tọa độ của H thỏa mãn phương trình tổng quát của đường thẳng AL. Do đó HL đi qua A. Ta kết thúc chứng minh. Chú ý. Trong [2] cũng có một lời giải thuần túy hình học cho Định lý 2. 3. Kết luận Tác giả coi bài viết trong [1] là công trình về hình học đầu tiên được tác giả công bố quốc tế. Tác giả rất tâm đắc với phép dựng Tỷ số Vàng này. Như trong [1] cũng đã nói, phép dựng này đã gợi nhớ lại phép dựng Tỷ số Vàng kinh điển của nghệ sĩ người Mỹ George Phillips Odom Jr, xem [5]. Trên cấu hình này (bao gồm cả phép dựng hình vuông nội tiếp) còn có nhiều bài toán mới và thú vị để khai thác, các bạn có thể tham khảo một vài bài toán như vậy ở trong [3, 4, 7]. Lời cuối cùng, tác giả muốn nói lời cảm ơn chân thành tới Giáo Sư Paul Yiu từ đại học Florida Atlantic của Mỹ. Ông như người bạn lớn, người thầy của tác giả, người đã giúp tác giả đặt những viên gạch đầu tiên để đưa những ý tưởng hình học của mình ra thế giới. 43 Tạp chí Epsilon, Số 23, 06/2023 Tài liệu [1] T. Q. Hung, The golden section in the inscribed square of an isosceles right triangle, Forum Geom., 15(2015), 91–92. [2] T. Q. Hung, BMoEG–II Mock Geometry Olympiad, Forum AoPS (2022), available at https: //artofproblemsolving.com/community/c594864t1273694f594864h2948721. [3] T. Q. Hung, Concurrent lines with a side of a square, Forum AoPS (2023), available at https://artofproblemsolving.com/community/c374081h3049737. [4] T. Q. Hung, Radical axis with the Lester circle, Forum AoPS (2023), available at https://artofproblemsolving.com/community/c374081h3051195. [5] T. Q. Hung and F. V. Lamoen , Odom’s Triangle, Int. J. Comput. Discov. Math. (2021), 68–77. [6] Trần Quang Hùng, Đề thi Olympic toán cho học sinh phần hình học, Hội Toán học Việt Nam (VMS) (2023). [7] Trần Quang Hùng, Ứng dụng của một số định lý hình học kinh điển, Kỷ yếu Trường Đông cho học sinh THPT chuyên, Khoa Toan–Cơ–Tin học ĐHKHTN–ĐHQGHN, Hà Nội 2022. [8] Trần Nam Dũng (chủ biên), Sách giáo khoa Toán 10 tập 2 (bộ sách chân trời sáng tạo), NXBGD 2023. 44 Tạp chí Epsilon, Số 23, 06/2023 Đồ thị của hàm số đa thức Lê Phúc Lữ (ĐH KHTN TPHCM) Trần Nguyễn Thanh Danh (PTNK TPHCM) Giới thiệu. Ta bắt đầu từ câu hỏi: tại sao phần mềm Geogebra có chức năng dựng đường conic đi qua 5 điểm và đồ thị conic sẽ luôn dựng được nếu trong 5 điểm đã xét, không có 3 điểm thẳng hàng? Mong rằng qua bài viết này, bạn đọc có thể tự trả lời được câu hỏi đó. 45 Tạp chí Epsilon, Số 23, 06/2023 1. Các bài toán Ta xuất phát từ bài toán nhẹ nhàng như sau: Bài toán 1 Cho hàm số f(x) = x2 − 2x, tìm điều kiện của tham số m để f(f(. . .(x)...)) | {z } 2022 có 22022 nghiệm thực phân biệt. = m Lời giải Ta vẽ đồ thị C của f(x) như bên dưới và đi biện luận từ f(x) = m (*). Để phương trình này có hai nghiệm phân biệt thì đường thẳng nằm ngang y = m phải cắt đồ thị C tại hai điểm, dựa vào C thì dễ thấy m > −1. Ta đặt hai nghiệm tương ứng là x1, x2. Lại xét tiếp f(f(x)) = m, để phương trình này có 22 = 4 nghiệm thì trước đó, (∗) phải có 2 nghiệm phân biệt, đặt là x1, x2 như trên. Ta viết lại f(f(x)) = m ⇔ (f(x) − x1)(f(x) − x2) = 0. Mỗi phương trình f(x) − xi = 0, ∀i = 1, 2 lại phải có 2 nghiệm phân biệt nên ta coi các số xi này có vai trò như tham số m ở trên và điều kiện cho các số này là xi > −1. Tiếp tục dựa vào đồ thị, ta thấy (∗) có hai nghiệm phân biệt xi > −1 khi −1 < m < 3. Nếu xét tiếp f(f(f(x))) = m có 23 = 8 nghiệm thì tương tự trên, cần tìm m để (∗) có hai nghiệm xi phân biệt thỏa mãn −1 < xi < 3. Đây là điểm mấu chốt của bài toán, ta quan sát đồ thị và thấy rằng, đường thẳng y = 3 cắt C tại (−1; 3) và (3; 3). Vì thế với điều kiện −1 < m < 3 thì (∗) cũng sẽ có hai nghiệm phân biệt xi cũng thỏa mãn −1 < xi < 3. Với sự “may mắn” này, điều kiện cho các phương trình chứa hàm hợp nhiều lần hơn sẽ không thay 46 Tạp chí Epsilon, Số 23, 06/2023 đổi. Vì thế nên để f(f(. . .(x). . .)) | {z } k = m, k ≥ 3 có 2k nghiệm phân biệt thì điều kiện là −1 < m < 3. Có một câu hỏi đặt ra là với điều kiện nào của a, b, c thì tam thức ax2 + bx + c có đặc điểm tương tự trên? Tiếp theo, ta xét một bài toán khác cũng rất thú vị về tam thức bậc hai. Bài toán 2 Cho tam thức bậc hai hệ số thực f(x) = ax2 + bx + c với a ̸= 0. 1. Biết rằng f(f(x)) = x có nghiệm thực duy nhất x = x0. Tính giá trị của f′(x0). 2. Giả sử rằng đồ thị hàm số y = f(x) và x = f(y) cắt nhau tại bốn điểm phân biệt tạo thành tứ giác ABCD. Chứng minh rằng AC⊥BD và ABCD không phải là hình thang. Lời giải 1) Dưới đây, ta xét ba cách giải cho bài toán này: Cách 1. Theo đề bài, f(x) = x phải có nghiệm duy nhất vì: • Nếu f(x) = x vô nghiệm thì phải có f(x) > x, ∀x hoặc f(x) < x, ∀x. Trường hợp thứ nhất sẽ kéo theo f(f(x)) > f(x) > x, ∀x dẫn đến phương trình ban đầu vô nghiệm. Trường hợp sau cũng tương tự. • Nếu f(x) = x có 2 nghiệm phân biệt là x1, x2 thì f(f(x1)) = f(x1) = x1 và f(f(x2)) = x2 nên f(f(x)) = x có ít nhất hai nghiệm, cũng không thỏa mãn. Do đó, f(x) = x có nghiệm duy nhất, tức là nghiệm kép, cũng chính là nghiệm x0 ban đầu. Suy ra f(x) − x = a(x − x0)2. Đạo hàm hai vế được f′(x) − 1 = 2a(x − x0) nên có ngay f′(x0) = 1. Cách 2. Ta có kết quả tổng quát hơn: nếu đa thức P(x) bậc chẵn có nghiệm duy nhất x0 thì đó cũng là nghiệm của P′(x). Thật vậy, theo định lý Bezout 47 Tạp chí Epsilon, Số 23, 06/2023 thì P(x) = (x − x0)Q(x) với Q(x) là đa thức bậc lẻ. Khi đó, Q(x) phải còn nghiệm nữa, và nghiệm đó phải trùng với x0 nên ta đặt tiếp P(x) = (x − x0)2H(x). Từ đây đạo hàm hai vế được P′(x) = (x − x0)2H′(x) + 2(x − x0)H(x) nên tiếp tục có nghiệm x = x0. Cách 3. Gọi (C) là đồ thị của y = f(x) và ta chia nó làm hai phần (C1),(C2) lần lượt nằm bên trái – phải của trục đối xứng. Khi đó, đồ thị (C′) của x = f(y) sẽ gồm hai phần (C′1),(C′2) lần lượt là đối xứng của (C1),(C2) qua đường thẳng (d) : y = x. Để f(f(x)) = x có nghiệm duy nhất thì (C),(C′) phải tiếp xúc nhau tại một điểm I nào đó. Do tính đối xứng trục thì I ∈ (d) nên (d) là tiếp tuyến chung của (C),(C′). Cuối cùng, ta biết rằng để tìm hệ số góc của tiếp tuyến, ta xét đạo hàm tại điểm đó, vì thế nên f′(x0) chính là hệ số góc của tiếp tuyến (d), cũng chính là 1. 2) Không mất tính tổng quát, ta giả sử a > 0. Ta biểu diễn như hình vẽ bên dưới, trong đó A = (C1) ∩ (C′2), B = (C1) ∩ (C′1), C = (C2) ∩ (C′1), D = (C2) ∩ (C′2). Khi đó, do tính đối xứng trục (d) thì A, C đối xứng nhau qua (d) nên AC⊥d. Ngoài ra, cũng có B, D ∈ d nên ta có ngay AC⊥BD. Tiếp theo, vì A, B ∈ (C1) là nhánh nghịch biến của đồ thị nên hệ số góc của AB là âm; tương tự hệ số góc của CD là dương nên AB, CD không thể song song nhau. Do BC, AD đối xứng với AB, CD qua trục (d) nên hai đường này cũng không song song. Vậy ta luôn có AC⊥BD và tứ giác ABCD này không phải là hình thang. Bài toán tiếp theo được tổng quát từ đề thi IMO Shortlist cách đây nhiều năm. Vai trò của đồ thị trong bài toán này là khá nhiều, tuy đó không phải là đồ thị của đa thức mà là phân thức. Bài toán 3 Cho đa thức hệ số thực P(x) bậc n ≥ 2 có n nghiệm thực phân biệt a1 < a2 < ... < an.Với số k > 0 cho trước, gọi D là tổng độ dài khoảng nghiệm của P′(x) P(x) ≥ k. Chứng minh D =nk. Lời giải Theo định lý Bezout thì ta có thể viết P(x) = a(x − a1)(x − a2)· · ·(x − an), khi đóP′(x) P(x)=1 x − a1+1 x − a2+ · · · +1 x − an, đặt là f(x). Ta quy về xét bất phương trình f(x) ≥ k. Ta có f′(x) = −Pn i=1 1 (x−ai)2 < 0 nên hàm số nghịch biến trên từng khoảng xác định. Ta có thể vẽ bảng biến thiên hoặc đồ thị của hàm số như hình bên dưới: 48 Tạp chí Epsilon, Số 23, 06/2023 Không mất tính tổng quát, giả sử k > 0 (trường hợp k < 0 thực hiện tương tự). Khi đó, trên mỗi khoảng (ai, ai+1), i = 1, 2, . . . , n − 1 và (an; +∞) thì f(x) = k có đúng một nghiệm. Đặt các nghiệm đó theo thứ tự là b1, b2, . . . , bn. Khi đó, các khoảng nghiệm là (ai, bi) với 1 ≤ i ≤ n và tổng độ dài các khoảng nghiệm là D =Xn i=1 (bi − ai) = Xn i=1 bi −Xn i=1 ai. Các nghiệm bi, 1 ≤ i ≤ n ở trên cũng đều là nghiệm của P′(x) P(x)= k ⇔ kP(x) − P′(x) = 0. Đặt P(x) = cnxn + cn−1xn−1 + · · · + c1x + c0 thì P′(x) = cnnxn−1 + cn−1(n − 1)xn−2 + · · · + c1 thay vào cnn · xn−1 + cn−1(n − 1)xn−2 + · · · + c1 = k(cnxn + cn−1xn−1 + · · · + c1x + c0) hay kcnxn + (kcn−1 − cnn)xn−1 + · · · + (kc1 − 2c2)x + kc0 = 0. Theo định lý Viete thì b1 + b2 + · · · + bn = −kcn−1 − cnn kcn =nk−cn−1 cn=nk+ (a1 + a2 + · · · + an). Từ đó ta có ngay D =nk. Tiếp theo, ta xét bài toán rất thú vị trong đề kiểm tra trường Đông 2022 49 Tạp chí Epsilon, Số 23, 06/2023 Bài toán 4 (trường Đông Vinh 2022) Với n nguyên dương, xét đa thức hệ số thực P(x) bậc n,monic sao cho tồn tại các số thực r, s, t phân biệt có tổng là −2023 thỏa mãn P(k) ∈ {r, s, t} với mọi k = 1, 2, 3, . . . , 3n − 1, 3n. 1. Với n = 2, tìm tất cả các đa thức P(x) thỏa mãn đề bài. 2. Hỏi có tồn tại hay không số n ≥ 3 để có đa thức P(x) thỏa mãn đề bài? Vì sao? Lời giải 1) Ứng với n = 2, ta có P(k) ∈ {r, s, t} với k = 1, 2, 3, 4, 5, 6. Do deg P = 2 nên có không quá 2 số k để P(k) = r. Tương tự với P(k) = s, P(k) = t. Từ đó suy ra mỗi phương trình trên phải có đúng 2 nghiệm. Chú ý rằng cả ba phương trình đều có chung hệ số x2 và x nên tổng các nghiệm của các phương trình là bằng nhau. Mặt khác, tất cả các nghiệm của chúng là 1, 2, 3, 4, 5, 6 có tổng bằng 21, từ đó suy ra mỗi phương trình sẽ có tổng các nghiệm là 7. Khi đó, 6 nghiệm ở trên sẽ chia ra các cặp (1, 6),(2, 5),(3, 4). Ta có thể giả sử  từ đó suy ra P(x) − r = (x − 1)(x − 6) P(x) − s = (x − 2)(x − 5) , P(x) − t = (x − 3)(x − 4) 3P(x) − (r + s + t) = (x − 1)(x − 6) + (x − 2)(x − 5) + (x − 3)(x − 4) hay 3P(x) + 2023 = 3x2 − 21x + 28. Từ đó tìm được P(x) = x2 − 7x − 665 và r, s, t lần lượt là −671, −675, −677. 2) Giả sử tồn tại P(x) bậc n ≥ 3 thỏa mãn đề bài, không mất tính tổng quát giả sử r < s < t. Theo lập luận ở trên, mỗi phương trình P(x) − r, P(x) − s, P(x) − t sẽ có đúng n nghiệm phân biệt lấy từ {1, 2, 3, . . . , 3n}. Ngoài ra, theo định lý Viete, do mỗi phương trình đều chung nhau ít nhất là ba hệ số đầu tiên tổng các nghiệm và tổng bình phương các nghiệm của chúng phải đều bằng nhau. 50 Tạp chí Epsilon, Số 23, 06/2023 Xét hàm số f(x) = P(x) − r có n nghiệm phân biệt nên theo định lý Rolle thì f′(x) = P′(x) phải có n − 1 nghiệm phân biệt, ký hiệu là c1 < c2 < . . . < cn−1. Phương trình P(x) = r sẽ có nghiệm duy nhất trên từng khoảng (−∞; c1),(c1, c2), . . . ,(cn−1, +∞). Tương tự với P(x) = s, P(x) = t. Ta có hai trường hợp sau a) Nếu n lẻ thì dễ thấy ở khoảng đầu tiên, P(x) đồng biến nên P(x) = r, P(x) = s, P(x) = t sẽ lần lượt nhận các nghiệm 1, 2, 3. Ở khoảng tiếp theo, hàm số nghịch biến nên chúng sẽ lần lượt nhận các nghiệm 6, 5, 4, và cứ như thế, đến khoảng cuối cùng sẽ là 3n − 2, 3n − 1, 3n. Khi đó, dễ thấy tổng các nghiệm của ba phương trình trong n − 1 khoảng đầu là bằng nhau, riêng khoảng cuối thì mỗi phương trình nhận một nghiệm khác nhau nên tổng các nghiệm của chúng là khác nhau, không thỏa mãn. b) Nếu n chẵn, đặt n = 2m thì ba phương trình sẽ có các nghiệm là 1, 2, 3, . . . , 6m và tương tự lập luận trên, P(x) = r sẽ có các nghiệm là {1, 6, 7, 12, . . . , 6m − 5, 6m}. Tổng bình phương các số này sẽ là Xm k=1 (6k − 5)2 + (6k)2 =Xm k=1 (72k2 − 60k + 25) 6− 60 ·m(m + 1) = 72 ·m(m + 1)(2m + 1) 2+ 25m = 12m(m + 1)(2m + 1) − 30m(m + 1) + 25m = m(24m2 + 6m + 7). Mặt khác, tổng bình phương tất cả 6m số là 6m(6m + 1)(12m + 1) 6= m(6m + 1)(12m + 1) nên mỗi phương trình phải có tổng bình phương các nghiệm là 13giá trị này. Suy ra 3m(24m2 + 6m + 7) = m(6m + 1)(12m + 1) hay 72m2 + 18m + 21 = 72m2 + 18m + 1, vô lý. Điều này cho thấy trong mọi trường hợp, ta không thể có đa thức P(x) thỏa mãn đề bài. Bài toán trên có thể nói là kết hợp của các bài toán sau đây: • Tìm tất cả các đa thức P(x) bậc ba monic sao cho |P(1)| = |P(2)| = |P(3)| = |P(5)| = |P(6)| = |P(7)| . • Xét các số nguyên dương m, n và giả sử tồn tại P(x) hệ số nguyên: P(x1) = · · · = P(xm) = 61 và P(y1) = · · · = P(yn) = 2020, với x1, x2, . . . , xm, y1, y2, . . . , yn là các số nguyên đôi một phân biệt. Tìm giá trị lớn nhất của mn. 51 Tạp chí Epsilon, Số 23, 06/2023 • Tìm tất cả các đa thức P(x), Q(x) hệ số nguyên sao cho P(Q(x)) = (x − 1)(x − 2)· · ·(x − 9) với mọi x ∈ R. Ở câu b) của bài toán, việc dùng đồ thị để minh họa rõ thứ tự sắp xếp của các nghiệm là rất quan trọng, giúp ta có thể chỉ rõ được các nghiệm ban đầu được phân hoạch ra như thế nào và từ đó dùng định lý Bezout, Viete. Bài toán 5 (Arab Saudi TST 2015) Cho tập hợp S = {(a, b)|a ̸= b, 1 ≤ a ≤ 4, 1 ≤ b ≤ 4} là các cặp số nguyên. Xét đa thức hai biến f(x, y) hệ số nguyên sao cho f(a, b) = 0, ∀(a, b) ∈ S. 1. Tìm giá trị nhỏ nhất của bậc của đa thức này. 2. Giả sử f0(x, y) là một đa thức có bậc nhỏ nhất. Chứng minh rằng f0 3 2;5+√6 2 = 0. Lời giải Bài toán này có nét tương tự đa thức nguyên tối tiểu, nếu lập luận theo hướng đại số trực tiếp thì vẫn được nhưng khá rắc rối. Trong phần này, ta sẽ biểu diễn các điểm lên mặt phẳng tọa độ, quy việc tìm đa thức về việc xác định đồ thị đi qua được tất cả các điểm này. Trước hết, ta thấy rằng nếu chỉ dùng đường thẳng thì cần ít nhất 4 đường mới đi qua hết 12 điểm này, nếu chỉ có 1, 2, hoặc 3 đường thì không đủ. Ta xét các trường hợp sau: 1. Nếu deg f(x, y) = 1 thì ta có đường thẳng dạng ax + by + c = 0 nên không thỏa mãn. 2. Nếu deg f(x, y) = 2 thì loại trường hợp tích của hai đa thức bậc nhất, nếu đây là đa thức bậc hai thì phương trình sẽ có dạng f(x, y) = ax2 + bxy + cy2 + dx + ey + g. Ta sẽ chứng minh conic dạng này sẽ không đi qua ba điểm thẳng hàng. Giả sử rằng nó đi qua được ba điểm P, Q, R cùng thuộc đường thẳng mx + ny + p = 0. Giả sử m ̸= 0, ta coi đây là đa thức bậc nhất theo x 52 Tạp chí Epsilon, Số 23, 06/2023 và coi f(x, y) là đa thức bậc hai theo biến x thì thực hiện phép chia f(x, y) = (mx + ny + p)g(x, y) + Ay2 + By + C Vì f(xP , yP ) = f(xQ, yQ) = f(xR, yR) = 0 nên yP , yQ, yR là ba nghiệm phân biệt của Ay2 + By + C nên rõ ràng phải có A = B = C = 0.Điều này kéo theo f(x, y) có thể phân tích thành tích của hai hàm số bậc nhất theo biến x, y, không thể là đường conic không suy biến được. Trở lại bài toán, ta thấy trong 12 điểm đã cho, có nhiều bộ ba điểm thẳng hàng nên không thể có conic không suy biến cùng lúc đi qua tất cả các điểm đó. 3. Nếu deg f(x, y) = 3, ta chỉ xét trường hợp một đường thẳng và một conic bậc hai. Quan sát đồ thị, ta thấy rằng: đường thẳng qua được tối đa 4 điểm; còn theo phần (2) ở trên thì ở mỗi tung độ y = 1, 2, 3, 4, đường conic sẽ qua tối đa 2 điểm nên tổng cộng qua được tối đa 8 điểm. Do đó, tổng cộng đồ thị qua tối đa 12 điểm và dấu bằng phải xảy ra. Ta vẽ được duy nhất mô hình gồm một đường thẳng và một đường tròn như bên dưới: Khi đó ta có được f0(x, y) = k(x + y − 5)(x2 + y2 − 5x − 5y + 10) và có ngay f 3 2;5+√6 2 = 0. Cuối cùng, ta xét hai bài toán rất thú vị về ý nghĩa hình học của các bài toán về đa thức, bài đầu tiên trong đề chọn đội tuyển của Iran, còn bài sau chúng tôi tương tự hóa lại từ bài này. Bài toán 6 Cho P(x) là đa thức hệ số thực khác hằng thỏa mãn: tồn tại vô số cặp số nguyên m, n sao cho P(m) + P(n) = 0. Chứng minh rằng đồ thị của hàm số y = P(x) có tâm đối xứng. Lời giải Ta biết rằng P(x) có tâm đối xứng là (x0, y0) nếu như P(x + x0) + P(−x + x0) = 2y0 với mọi x ∈ R. Nói cách khác, nếu đặt Q(x) = P(x + x0) − y0 thì Q(x) + 53 Tạp chí Epsilon, Số 23, 06/2023 Q(−x) = 0 hay Q là hàm số lẻ. Như thế, mấu chốt của bài toán này là chỉ ra trong vô số cặp số nguyên đã cho, phải có vô số cặp số nguyên có tổng là hằng số nào đó. Không mất tính tổng quát, ta có thể giả sử rằng P(x) monic, vì nếu P(x) thỏa mãn thì P(x)/c cũng thỏa mãn. Nếu như deg P(x) chẵn thì với x có giá trị tuyệt đối đủ lớn, P(x) > 0. Cụ thể là tồn tại các số thực p, q sao cho p < 0 < q mà P(x) > 0, ∀x /∈ [p; q]. Xét S = {s ∈ Z|s ∈ [p; q]} thì rõ ràng S hữu hạn. Xét cặp số nguyên (m, n) sao cho P(m) + P(n) = 0 thì giả sử P(n) ≤ 0, kéo theo n ∈ S.Như thế, với mọi cặp số nguyên (m, n) thỏa mãn đề bài thì có ít nhất một số sẽ thuộc S. Vì tính hữu hạn của S nên phải có một số s0 ∈ S ứng với vô số cặp (m, s0) mà P(m) = −P(s0), kéo theo P(x) là đa thức hằng, không thỏa mãn. Như vậy, deg P(x) lẻ. Bây giờ chú ý rằng kể từ một số x đủ lớn nào đó thì P(x) đơn điệu tăng khi x → +∞ (ta chỉ cần chọn x lớn hơn điểm cực trị lớn nhất của P(x) là xong). Hơn nữa, với mỗi số nguyên n, tồn tại hữu hạn các số nguyên m sao cho P(m) = −P(n) (vì đa thức chỉ có thể nhận một giá trị nào đó ở hữu hạn điểm, không vượt quá bậc của nó). Từ đó, ta thấy rằng với mọi số thực C đủ lớn, tồn tại cặp số m, n ∈ Z sao cho P(m) + P(n) = 0, trong đó m và n khác dấu và có trị tuyệt đối lớn hơn C. (∗) Giả sử rằng deg P(x) = k lẻ và đặt P(x) = xk + axk−1 + H(x) với deg H < k − 1. Tiếp theo, dễ dàng chọn được số thực d sao cho P(x − d) khuyết hệ số của bậc k − 1. Thật vậy, P(x−d) = (x − d)k+a(x − d)k−1+H(x−d) = xk−kdxk−1+axk−1+H(x−d) và như thế, ta chỉ cần chọn d =ak. Bây giờ ta chứng minh rằng điểm (d; 0) chính là tâm đối xứng của đồ thị hàm số y = P(x). Đặt P(x − d) = Q(x), ta quy về chứng minh rằng Q(x) = −Q(−x), ∀x ∈ R. Như thế, Q(x) = xk + bxk−2 + H(x − d) và tồn tại vô hạn cặp số m, n sao cho Q(m) +Q(n) = 0 và m −d, n−d là số nguyên. Theo (∗), ta chọn nghiệm có giá trị tuyệt đối đủ lớn với m > 0, n < 0. Bây giờ giả sử |m| < |n|, giả sử 54 Tạp chí Epsilon, Số 23, 06/2023 n = −m − c với c ∈ Z+. Khi đó Q(m) + Q(n) = Q(m) + Q(−m − c) = mk + bmk−2 + H(m) + (−m − c)k + b(−m − c)k−2 + H(−m − c) = −kc · mk−1 + R(m), trong đó deg R(x) ≤ k −2. Nếu m đủ lớn, số kc ·mk−1sẽ lớn hơn nhiều so với |R(m)| và như vậy tổng −kc · mk−1 + R(m) sẽ nhỏ hơn 0. Tương tự, không thể có |m| > |n| với |m| , |n| đủ lớn. Do đó, tồn tại vô số các số m sao cho Q(m) +Q(−m) = 0, tức là đa thức Q(x) +Q(−x) có vô số nghiệm. Điều này có thể xảy ra khi Q(x) + Q(−x) = 0, ∀x ∈ R và như thế, đa thức Q(x) là hàm số lẻ và có đồ thị đối xứng qua điểm (0; 0). Vậy đồ thị y = P(x) đối xứng qua điểm (d; 0). Bài toán 7 Cho P(x) là đa thức hệ số thực khác hằng thỏa mãn: tồn tại vô số cặp số nguyên m ̸= n sao cho P(m) = P(n). Chứng minh rằng đồ thị của hàm số y = P(x) có trục đối xứng. Lời giải Bài toán này nói chung giống bài trên, trong lời giải, ta chỉ cần đổi dấu cho P(x) tại một số vị trí thích hợp. Ta biết rằng P(x) có trục đối xứng là x = x0 nếu như P(x + x0) = P(−x + x0) với mọi x ∈ R. Nói cách khác, nếu đặt Q(x) = P(x + x0) thì Q(x) = Q(−x) hay Q là hàm số chẵn. Không mất tính tổng quát, ta cũng giả sử P(x) monic. Nếu deg P(x) lẻ thì rõ ràng lim x→−∞P(x) = −∞, lim x→+∞P(x) = +∞. Như thế, có thể chọn x0 đủ lớn thì P(x) sẽ đồng biến trên (x0; +∞) và P(x) = y0 luôn có nghiệm duy nhất với mọi y0 > P(x0). Do đó, không thể tồn tại cặp số m ̸= n như đề bài. Do đó deg P(x) chẵn. Xét deg P(x) chẵn thì tương tự trên, với mọi số thực C đủ lớn, tồn tại cặp số m, n ∈ Z sao cho P(m) = P(n) trong đó m và n khác dấu và có trị tuyệt đối lớn hơn C. (∗) Đặt P(x) = xk +axk−1+H(x) với k chẵn, deg H < k−1. Chọn d để P(x−d) khuyết bậc k − 1. Ta chứng minh rằng đường thẳng x = d chính là trục đối xứng của đồ thị hàm số y = P(x). 55 Tạp chí Epsilon, Số 23, 06/2023 Đặt P(x − d) = Q(x) thì Q(x) = xk + bxk−2 + H(x − d) và tồn tại vô hạn m, n sao cho Q(m) = Q(n) và m − d, n − d đều nguyên. Theo (∗), ta chọn nghiệm có giá trị tuyệt đối đủ lớn với m > 0, n < 0. Ta cũng sẽ đi chứng minh m = −n. Bây giờ giả sử |m| < |n|, giả sử n = −m − c với c ∈ Z+. Khi đó Q(m) − Q(n) = Q(m) − Q(−m − c) = mk + bmk−2 + H(m) − (−m − c)k − b(−m − c)k−2 − H(−m − c) = −kc · mk−1 + R(m), trong đó deg R(x) ≤ k − 2. Đến đây sẽ có vô lý. Tương tự nếu |m| > |n| cũng có vô lý, và vì thế nên m+n = 0. Vậy nên Q(x) = Q(−x) với vô số x nguyên, kéo theo Q(x) = Q(−x), ∀x ∈ R nên Q(x) có trục đối xứng là x = 0, và P(x) có trục đối xứng là đường thẳng x = d. 2. Bài tập rèn luyện. Bài toán 8 (VMO 2018). Trong mặt phẳng tọa độ Oxy, cho C là độ thị của hàm số y =√3x2. Một đường thẳng (d) thay đổi và cắt C tại ba điểm có hoành độ phân biệt lần lượt là x1, x2, x3. 1. Chứng minh rằng đại lượng sx1x2 sx2x3 sx3x1 3 x22 là hằng số. x23+ 3 x21+ 3 r 2. Chứng minh rằng 3 x21 r x22 r x23 x2x3+3 x3x1+3 x1x2< −154. Bài toán 9 (Bài toán friendly circles). Cho các số thực a, b, c đôi một phân biệt và khác 0. Chứng minh rằng ba đường tròn sau khi vẽ trong mặt phẳng tọa độ Oxy thì luôn có ít nhất hai điểm chung (x − a)2 + (y − b)2 = c2 (x − b)2 + (y − c)2 = a2 (x − c)2 + (y − a)2 = b2 (điểm chung được hiểu là giao điểm của ít nhất hai trong ba đường tròn trên). 56 Tạp chí Epsilon, Số 23, 06/2023 Bài toán 10 (AIME 2022). Cho a, b, x, y là các số thực (trong đó a > 4, b > 1) thỏa mãn a2 − 16=(x − 20)2 x2 a2 +y2 b2 − 1+(y − 11)2 b2 = 1. Tìm giá trị nhỏ nhất của T = a + b. Bài toán 11 (Đề THPT QG 2019). Tìm điều kiện của m để phương trình sau có bốn nghiệm phân biệt x − 2 x − 1+x − 1 x+x x + 1+x + 1 x + 2= |x + 1| − x − m. Tài liệu [1] Nguyễn Mạc Nam Trung, Tài liệu chọn lọc trường Đông Toán học miền Nam, 2020. [2] IMO Booklet của Arab Saudi 2015. [3] Phùng Hồ Hải, Bài giảng ở Trường Đông Titan miền Nam, 2022. [4] https://mathscope.org/showthread.php?t=35623. [5] https://artofproblemsolving.com/community/c5h2782937p24447217. [6] https://en.wikipedia.org/wiki/Five_points_determine_a_conic. 57 Tạp chí Epsilon, Số 23, 06/2023 Bài Toán Xây Dựng Đa Thức Đào Xuân Luyện Trường THPT chuyên Lê Quý Đôn, Bình Định Các bài toán liên quan đến đa thức thường xuất hiện trong các đề thi học sinh giỏi các cấp, thi chọn đội tuyển học sinh thi học sinh giỏi quốc gia của các tỉnh thành trong cả nước, thi olympic các cấp và thi học sinh giỏi quốc gia cũng như trong cuộc thi IMO. Trong quá trình giảng dạy, tác giả đã biên soạn tài liệu để giảng dạy, có bổ sung và sáng tác thêm. Các tài liệu tham khảo về vấn đề hiện nay đã có nhưng chưa hình thành nên một chủ đè trong giải một số dạng toán về đa thức. Nhằm trang bị thêm tài liệu giảng dạy và giúp cho các bạn đồng nghiệp, các học sinh chuẩn bị thi học sinh giỏi và những ai quan tâm đến vấn đề tham khảo, tác giả viết chủ đề “Bài toán xây dựng đa thức” nhằm giúp học sinh trực tiếp giảng dạy và các đồng nghiệp có nguồn để tham khảo và những người đọc quan tâm đến vấn đề hiểu tốt hơn và có cái nhìn tốt hơn về lý thuyết về đa thức cũng như ứng dụng nó. 58 Tạp chí Epsilon, Số 23, 06/2023 1. Một số kết quả thường sử dụng trong bài viết Ngoài phần lý thuyết cơ bản về đa thức như định nghĩa, các khái niệm như bậc, nghiệm, định lý Viet, định lý Bezout, định lý cơ bản của đại số, tính liên tục, tính khả vi,... ở đây bổ sung thêm một số kết quả sau. Định lý 1 Giả sử A là một trường, α ∈ A, f(x) ∈ A [x]. Dư của phép chia f(x) cho là f(α). Định lý 2 Ta có α là nghiệm của f(x) khi và chỉ khi f(x) chia hết cho (x − α). Giả sử A là một trường, α ∈ A, f(x) ∈ A [x] và m là một số tự nhiên lớn hơn hoặc bằng 1, α là nghiệm bội cấp m của f(x) nếu và chỉ nếu f(x) chia hết cho (x − α)m và f(x) không chia hết cho (x − α)m+1. Trong trường hợp m = 1 người ta còn gọi α là nghiệm đơn, m = 2, α gọi là nghiệm kép. Người ta coi một đa thức có một nghiệm bội cấp m như một đa thức có m nghiệm trùng nhau. Định lý 3 (Định lí Viette) (a) Với an ̸= 0, giả sử phương trình anxn + an−1xn−1 + · · · + a1x + a0 = 0 (1) có n nghiệm x1, x2, . . . , xn thì  x1 + x2 + · · · + xn = −an−1 an x1x2 + x1x3 + · · · + xn−1xn =an−2 an  · · · x1x2 · · · xn = (−1)n a0 an (2) (b) Ngược lại nếu các số x1, x2, . . . , xn thoả mãn hệ trên thì chúng là nghiệm của phương trình (1). Hệ (2) có n thành phần và ở vế trái của thành phần thứ k có Cnksố hạng. Định lý 4 Một đa thức bậc n có không quá n nghiệm. Đa thức có vô số nghiệm là đa thức không. Nếu đa thức có bậc ≤ n mà nhận giá trị bằng nhau tại n + 1 giá trị khác nhau của đối số thì đa thức đó là đa thức hằng. 59 Tạp chí Epsilon, Số 23, 06/2023 Hai đa thức bậc ≤ n mà nhận n + 1 giá trị bằng nhau tại n + 1 giá trị khác nhau của đối số thì đồng nhất bằng nhau. Định lý 5 Mọi đa thức f(x) ∈ C [x] bậc n có đúng n nghiệm (tính cả bậc của nghiệm). Định lý 6 Mọi đa thức f(x) ∈ R [x] có bậc n và có hệ số chính (hệ tử cao nhất) an ̸= 0 đều có thể phân tích duy nhất thành nhân tử f(x) = anYm i=1 (x − αi)Yl k=1 x2 + bkx + ck , với αi, bk, ck ∈ , l, m, n ∈∗thỏa mãn n = m + 2l và b2k − 4ck < 0. 2. Một số bài toán Sau đây là một số bài toán sử dụng cách xây dựng đa thức để giải: Bài toán 1. Có tồn tại hay không một đa thức P(x) bậc 2023 sao cho P(x2 − 2022) chia hết cho P(x). Phân tích: Các bài toán về tồn tại đa thức thường hay xây dựng đa thức để thỏa mãn. Một trong những dạng đa thức thường hay sử dụng là đa thức dạng lũy thừa của một nhị thức bậc nhất. Trong bài này, chúng ta cũng xây dựng theo hướng đó, cụ thể là xây dựng đa thức có dạng P(x) = (x + a)2023. Lời giải Xét đa thức P(x) = (x + a)2023, khi đó P(x2−2022) = (x2 − 2022 + a)2023 =h(x + a)2 − 2a(x + a) + a2 + a − 2022i2023. Nếu chọn được a sao cho a2 + a − 2022 = 0, tức 2∨ a =−1 −√8089 a =−1 + √8089 2. thì P(x2 − 2022) = (x2 − a2)2023 = (x + a)2023(x − a)2023 chia hết cho P(x). Vậy P(x) = x +−1 + √8089 2 !2023 , hoặc P(x) = x +−1 −√8089 2 !2023 . Thỏa mãn điều kiện bài toán. Trong lời giải trên chúng ta tìm đa thức dưới dạng lũy thừa của một nhị thức bậc nhất, theo hướng suy nghĩ ta có thể tìm đa thức dưới dạng là tích của các 60 Tạp chí Epsilon, Số 23, 06/2023 đa thức bậc nhất. Theo nghĩ đó ta có lời giải tiếp theo như sau Lời giải Ta tìm đa thức dưới dạng 2023 P2023(x) = Y k=1 (x − ak), ak ∈, Ta có P2023(x2 − 2022) =2023 Y k=1 (x2 − 2022 − ak). (1) Nếu chọn ak sao cho (để tích (1) có dạng −2022 − ak = −a2k, (2) Yn k=1 (x2 − a2k) = Yn k=1 (x − ak)(x + ak)...2023 Y k=1 (x − ak), thì P2023(x2 − 2022) =2023 Y k=1 (x2 − a2k) =P2023(x)2023 Y k=1 (x + ak), chia hết cho P2023(x)). Tuy nhiên dễ dàng thấy rằng 2, hoặc ak =1 −√8089 2, thỏa (2). ak =1 + √8089 Bài toán 2. Chứng minh rằng với mỗi số thực a > 1 và mỗi n ∈ N∗, tồn tại đa thức P (x) = xn + a1xn−1 + · · · + an ∈ R [x] , thỏa mãn các điều kiện sau (i) |ai| = aki, ki ∈ N, i = 1, 2, . . . , n. (ii) Tất cả các nghiệm của đa thức P (x) đều là các số thực khác nhau. Ngoài ra, khi n chẵn thì số các nghiệm dương bằng số các nghiệm âm, khi n lẻ thì giá trị tuyệt đối của hiệu giữa số các nghiệm dương và số các nghiệm âm bằng 1. Phân tích: Bài toán cần xây dựng đa thức P(x) có các hệ số thỏa mãn một điều kiện cho trước và có đủ nghiệm thực theo bậc của nó. Chúng ta quy nạp theo bậc của nó. Vì hệ số cao nhất của đa thức là 1 và các hệ số thỏa mãn điều kiện (i) nên chúng ta xây dựng đa thức theo đa thức ban đầu bằng cách cộng thêm đơn thức bậc lớn hơn giả thiết quy nạp là 1 và để bảo đảm điều kiện hệ số thỏa (i) thì ta nhân đa thức giả thiết quy nạp vơí lũy thừa của a là ak, điều chỉnh mũ k để bảo đảm điều kiện (ii). 61 Lời giải Ta chứng minh bằng quy nạp theo n ∈ N∗: Tạp chí Epsilon, Số 23, 06/2023 - Trường hợp n = 1, lấy P (x) = x ± ak, khi n = 2, lấy P (x) = x2 + akx − al. - Giả sử kết quả đúng đến n ≥ 2. Xét đa thức Q (x) có hệ số bậc cao nhất bằng 1, có bậc 2n và có các nghiệm là x1 < x2 < · · · < xn < 0 < xn+1 < xn+2 < · · · < x2n. Do tính liên tục của Q (x) nên nó không đổi dấu trong các khoảng (−∞; x1), (xi; xi+1), (x2n; +∞) và lim x→±∞Q (x) = +∞ vì vậy tồn tại một dãy các các số thực y1 < x1 < y2 < x2 < · · · < yn < xn < yn+1 < 0 < xn+1 < yn+2 < xn+2 < · · · < x2n < y2n+1 đồng thời Q (yi) có dấu +, −, +, −, bắt đầu với Q (y1) > 0. Xét đa thức P (x) = x2n+1 + akQ (x), ở đây ta chọn k ∈ N đủ lớn sao cho P (yi) cùng dấu với Q (yi). Do bậc của P (x) là lẻ nên lim x→±∞P (x) = ±∞ và do tính liên tục của P (x) nên P (x) có n + 1 nghiệm âm và n nghiệm dương zi thỏa mãn −∞ < z1 < y1 < z2 < y2 < · · · < yn < zn+1 < yn+1 < 0 < zn+2 < yn+2 < · · · < z2n+1 < y2n+1. Suy ra điều phải chứng minh. - Trường hợp đa thức Q (x) có hệ số bậc cao nhất bằng 1, có bậc 2n + 1 thì xét đa thức P (x) = x2n+2 − akQ (x), và lập luận tương tự như trên ta cũng suy ra đa thức P (x) có n + 1 nghiệm âm và n + 1 nghiệm dương. Chứng minh hoàn tất Bài toán 3. Tìm tất cả đa thức P(x) ∈ R[x] thỏa mãn P2(x) = 3P x2 − 5 + 1, ∀x ∈ R. (3) Phân tích: Chúng ta nhận thấy nếu đa thức P(x) là hằng số thì nếu P(x) = b thì b là nghiệm của phương trình b2 = 3b + 1, ta dễ dàng tìm ra số b và đa thức P(x) = b, x ∈ R thỏa yêu cầu bài toán, tuy nhiên việc tìm đa thức có bậc dương (nếu có) đòi hỏi chúng ta cần phải viết dạng tổng quát của đa thức P(x) sao cho P(a) = b với a là số thực nào đó. Một cách tự nhiên ,ta cần chọn a sao cho a thỏa a = a2 − 5. Lời giải Trước hết chúng ta thấy x2 − 5 = x ⇔ "x =1−√21 2 x =1+√21 2 62 Tạp chí Epsilon, Số 23, 06/2023 Giả sử đa thức P(x) thỏa mãn yêu cầu bài toán. Trong (3) thay x = a ta có: P2(a) = 3P a2 − 5 + 1 ⇔ P2(a) − 3P (a) − 1 = 0 P (a) = 3 −√13  ⇔ 2 P (a) = 3 + √13 2 n3−√13 o Giả sử P (x) là đa thức khác hằng, khi đó P (x) ̸= b với b ∈ với mọi x ∈ R và b thỏa b2 − 3b − 1 = 0. Do P(a) = b nên ta viết P(x) dưới dạng P(x) = (x − a)n· Q(x) + b, Q(a) ̸= 0. Thay vào (3), ta được 2,3+√13 2 ((x − a)nQ(x) + b)2 = 3 h x2 − 5 − a n· Q x2 − 5 + bi+ 1, và (x − a)2n·Q2(x)+ 2b(x − a)n·Q(x)+b2 = 3 x2 − a − 5 nQ x2 − 5 + 3b+ 1 (4) Vì b2 = 3b + 1, a2 = a + 5 nên (4) được viết thành (x − a)2n· Q2(x) + 2b(x − a)n· Q(x) = 3 x2 − a2 nQ x2 − 5 , hay (x − a)n· Q2(x) + 2b · Q(x) = 3(x + a)n· Q x2 − 5 . Thay x = a, ta có 2b · Q(a) = 3(2a)n· Q a2 − 5 , 2b · Q(a) = 3(2a)n· Q(a), 2b = 3 · (2a)n. Hay 3±√13 = 3(1 ±√21)n, điều này không xảy ra với mọi số nguyên dương n. Do đó P(x) = b. Thử lại P(x) = b thỏa mãn yêu cầu bài toán. Vậy P(x) ≡3−√13 2hoặc P(x) ≡3+√13 2. Trong lời giải trên, việc chọn ra các số a là quan trọng và tìm ra số b để từ đó dựng nên đa thức P(x) có dạng P(x) = (x − a)n· Q(x) + b với n ∈ Z+, Q(a) ̸= 0. Số a có được là nghiệm của phương trình liên quan đến các biểu thức chứa x trong yêu cầu bài toán, đó là phương trình x2 − 3 = x và số b là giá trị của đa thức P(x) tại x = a. Sử dụng cách thức xử lý chúng ta có thể sáng tác nhiều bài toán tương tự. Bạn đọc có thể thử sức với bài toán sau: 63 Tạp chí Epsilon, Số 23, 06/2023 Tìm tất cả các đa thức P (x) với hệ số thực thỏa mãn với mọi số thực x sao cho P2(x) = 2P x2 − 3 + 1. Tương tự như trên, có 2 đa thức thỏa mãn yêu cầu đề bài là P (x) = 1 + √2 và P (x) = 1 −√2. Bài toán 4 (Romania 1990). Tìm tất cả các đa thức P ∈ [x] , thỏa mãn 2P(2x2 − 1) = [P(x)]2 − 2, ∀x ∈ . Phân tích: Chúng ta có thể giải quyết bài toán và trình bày lời giải tương tự như bài toán 3, ở đây ta xét thêm một cách trình bày lời giải của bài toán này, mặc dù về cơ bản vẫn giống về ý tưởng. Lời giải Giả sử P(x) ̸= P(1), đặt P(x) = (x − 1)nQ(x) + P(1), Q(1) ̸= 0, n ∈ . Khi đó, từ giả thiết suy ra 2n+1(x − 1)n(x + 1)nQ(2x2−1)+2P(1) = (x − 1)2n[Q(x)]2+2(x − 1)nQ(x)P(1)+[P(1)]2−2. Thay x = 1, suy ra 2f (1) = f2(1) − 2, và 2n+1(x + 1)nQ(2x2 − 1) = (x − 1)nQ2(x) + 2Q(x)P(1). Cho x = 1, ta được Q(1) · Lại cho x = 1 vào 22n+1 − P(1) = 0. (5) 2P(2x2 − 1) = P2(x) − 2, ∀x ∈, ta giải ra được P(1) = 1 ±√2 ∈/, nên từ (5) suy ra Q(1) = 0 (mâu thuẫn). Như vậy P(x) ≡ P(1) là đa thức hằng và P(x) = 1 ±√3. Bài toán 5. Tìm cặp đa thức hệ số nguyên P(x), Q(x) khác đa thức không sao cho P Q √2 + √17 + √19 √2 + √17 + √19 =√2 + √17. Phân tích: Để giải quyết bài toán ta tìm cách tách √19, khi đó nếu đặt a = √2 + √17 + √19 thì √2 + √17 = a −√19 và tìm cách xây dựng đa thức để loại bỏ √2+√17. Thông thường như vậy ta thường xây dựng trực tiếp đa thức tích các nhị thức bậc nhất để loại √2 + √17. 64 Tạp chí Epsilon, Số 23, 06/2023 Lời giải Đặt a =√2 + √17 + √19 ⇒√2 + √17 = a −√19. Xét đa thức H(x) = x −√2 −√17 −√19 x +√2 + √17 −√19 x +√2 −√17 −√19 x −√2 + √17 −√19 = x −√19 2− √2 + √17 2 x −√19 2− √2 −√17 2 = x −√19 4− 38 x −√19 2+ 225 = x4 − 4√19x3 + 114x2 − 76√19x + 361 − 38 x2 − 2√19x + 19 + 225 = x4 + 76x2 − 136 − 4x3√19. Đặt A(x) = x4 + 76x2 − 136, Q(x) = 4x3. Rõ ràng theo cách đặt thì H(a) = 0 ⇒ A(a) − Q(a)√19 = 0. Do đó ta cóA(a) Q(a)=√19 ⇒A(a) Q(a)= a − Suy ra√2 + √17 = a −A(a) √2 + √17 . Q(a)= a −a4 + 76a2 − 136 4a3 =3a4 − 7a2 + 136 4a3. Vì vậy, nếu chọn P(x) = 3x4 − 76x2 + 136, Q(x) = 4x3, thì ta có P Q √2 + √17 + √19 √2 + √17 + √19 =√2 + √17. Bằng cách tương tự, chúng ta có thể sáng tác nhiều bài toán khác Tìm cặp đa thức hệ số nguyên P(x), Q(x) khác đa thức không sao cho P Q √2 + √5 √2 + √5 =√5. Tìm cặp đa thức hệ số nguyên P(x), Q(x) khác đa thức không sao cho P Q √2 + √5 + √7 √2 + √5 + √7 =√2 + √7. Bài toán 6. Cho f(x) = ax2+bx+c ∈ R[x] là đa thức bậc hai thỏa mãn điều kiện f(x) ≥ 0, với mọi x ≥ 0. Chứng minh rằng tồn tại đa thức P(x) ∈ R[x] 65 Tạp chí Epsilon, Số 23, 06/2023 sao cho đa thức f(x) · P(x) có tất cả các hệ số đều không âm. Lời giải Do f(x) ≥ 0, ∀x ≥ 0 nên a > 0, c = f(0) ≥ 0, ∆f = b2 − 4ac ≤ 0. ◦ Nếu b ≥ 0 thì chọn P(x) = 1 thì ta được điều phải chứng minh. ◦ Nếu b < 0 thì ta tìm P(x) dưới dạng P(x) = (x + 1)n, n ≥ 2. Khi đó f(x) · P(x) = (ax2 + bx + c) · (x + 1)n = (ax2 + bx + c) ·Xn k=0 Cknxk = axn+2 + (b + an)xn+1 +Xn (aCk−2 n + cCkn)xk + (b + cn)x + c n + bCk−1 k=2 Như vậy, ta cần chọn n sao cho  b + an ≥ 0 (1) b + cn ≥ 0 (2)  aCk−2 n + bCk−1 n + cCkn ≥ 0, ∀k ≥ 2 (3) −b Nhận thấy với n > max thành a;−bc thì (1) và (2) thỏa mãn. Ta biến đổi (3) g(k) = (a−b+c)k2 −[a−b(n+ 2) +c(2n+ 3)]k +c(n+ 1)(n+ 2) ≥ 0, ∀k ≥ 2 Chú ý rằng hệ số của k2là a − b + c > 0 (do a > 0, b < 0, c ≥ 0), do dó để (3) đúng với mọi k ≥ 2, ta chọn n sao cho biệt thức ∆g của tam thức bậc hai g(k) không dương. Rõ ràng ∆g cũng là một tam thức bậc hai theo n có hệ số của n2là (b − 2c)2 − 4c(a − b + c) = b2 − 4ac ≤ 0. nên limn→∞∆g = −∞. Do đó với n đủ lớn thì ∆g ≤ 0. Từ đó suy ra tồn tại n đồng thời thỏa mãn (1), (2), (3), và ta thu được điều phải chứng minh. Bài toán 7. Cho đa thức: g(x) = anxn + an−1xn−1 + · · · + a1x + a0, n ≥ 3 thỏa mãn g(x) > 0 với mọi x > 0. Chứng minh rằng tồn tại số tự nhiên m sao cho đa thức Q(x) = g(x)(x + 1)m có các hệ số đều không âm. Lời giải Ta xét 2 trường hợp : ◦ Khi deg g(x) = n = 2k thì ta có thể phân tích g(x) = Yk i=1 (aix2 + bix + ci), 66 Tạp chí Epsilon, Số 23, 06/2023 trong đó aix2 + bix + ci > 0, ∀x > 0. Theo bài toán 6, với mỗi đa thức aix2 + bix + ci đều tồn tại số tự nhiên mi sao cho đa thức Qi(x) = (aix2 + bix + ci)(x + 1)mi có các hệ số đều không âm. Từ đó Yk i=1 Qi(x) = Yk i=1 (aix2 + bix + ci)(x + 1)mi = g(x)(x + 1)m1+···+mk có các hệ số đều không âm. Chọn m = m1 +m2 +· · ·+mk, ta được điều phải chứng minh. ◦ Khi deg g(x) = n = 2k + 1 thì g(x) có ít nhất một nghiệm không dương là −a, (a ≥ 0). Khi đó g(x) = (x + a)h(x), với h(x) > 0, ∀x > 0 và deg h(x) = 2k. Theo trường hợp trên, tồn tại số tự nhiên m sao cho h(x)(x + 1)m có các hệ số đều không âm, suy ra đa thức g(x)(x + 1)m cũng có các hệ số đều không âm. Bài toán 8. Cho 2n số thực đôi một khác nhau a1, a2, . . . , an và b1, b2, . . . , bn. Điền vào bảng n×n theo quy tắc ô (i; j) (hàng i và cột j) là số ai+bj . Chứng minh rằng nếu tích tất cả các số trên các cột bằng nhau thì tích các số ở mỗi hàng cũng bằng nhau. Phân tích: Yêu cầu bài toán là chứng minh nếu tích tất cả các số trên các cột bằng nhau thì tích các số ở mỗi hàng cũng bằng nhau nên nghĩ đến việc xây dựng đa thức có chứa các thừa số. Lời giải Xét đa thức P(x) = (x + a1)(x + a2)· · ·(x + an) Rõ ràng P(bj ), j = 1, n là tích các số ở cột thứ i. Theo giả thiết, ta có P(b1) = P(b2) = · · · = P(bn) = C = const. Do đó đa thức G(x) = P(x) − C là đa thức bậc n, hệ số cao nhất bằng 1 và có n nghiệm phân biệt b1, b2, . . . , bn. Suy ra G(x) = (x + a1)(x + a2)· · ·(x + an) − C = (x − b1)(x − b2)· · ·(x − bn). Lần lượt thay x = −ai, i = 1, n vào đẳng thức trên, ta được −C = (−aj − b1)(−aj − b2)· · ·(−aj − bn), hay (aj + b1)(aj + b2)· · ·(aj + bn) = (−1)n+1· C = const. Vế trái đẳng thức trên là tích các số ở hàng j. Từ đây dễ dàng suy ra điều phải chứng minh. Để xây dựng được đa thức như bài giải là vấn đề rất khó, người giải phải có những tư duy rất giỏi mới nghĩ đến được cách xây dựng này. 67 Tạp chí Epsilon, Số 23, 06/2023 Bài toán 9. Cho số nguyên m > 2. Chứng minh rằng với mọi số tự nhiên n ≥ 3 thì số m2n−1 − 1 m − 1− mn, luôn có một ước số dương có dạng ma + 1, với a là số tự nhiên. Lời giải Ta cóm2n−1 − 1 m − 1− mn =m2n−1 + mn − mn+1 − 1 m − 1. Đặt n + 1 = 2r· s, trong đó r, s ∈ N, r < n và s lẻ. Xét đa thức P(m) = m2n−1 + mn − mn+1 − 1 = mnhm2r(2n−r−s) − 1i− (m2r·s + 1). Do s và 2n−r − s đều lẻ nên P(m) chia hết cho m2r+ 1, hay P(m) = (m2r+ 1) · Q(m), trong đó Q(m) ∈ Z[x]. Mà P(1) = 0 nên Q(1) = 0, hay Q(m) = (m − 1) · H(m), trong đó H(m) ∈ Z[x]. Suy ra m2n−1 − 1 m − 1− mn =P(m) m − 1= (m2r+ 1) · H(m), từ đó suy ra điều phải chứng minh. Trong bài đã xây dựng đa thức để phân tích được số đã cho là tích của hai số nguyên dương, trong đó có một thừa số theo yêu cầu của đề bài. Bài toán 10 (Bài P118 – Tạp chí Pi). Cho số nguyên dương k và đa thức hệ số nguyên P(x) khác hằng và không có nghiệm nguyên. Tìm đa thức hệ số nguyên Q(x) thỏa mãn ≡ 0 (mod P(n)), ∀n ∈ N∗. Q Phân tích: n k - Theo yêu cầu bài toán, chúng ta cần nghĩ đến việc xây dựng đa thức Q(x) sao cho Q n k là một tích trong đó có chứa thừa số P(n). - Xuất phát từ hệ thức quen thuộc n k ≤nk< nk + 1, ta có thể xây dựng đa thức sau và kiểm tra nó thỏa yêu cầu bài toán Q(x) = P(kx) · P(kx + 1)· · · P(kx + k − 1). Lời giải Ta sẽ chứng minh Q(x) = P(kx) · P(kx + 1)· · · P(kx + k − 1) thỏa mãn yêu cầu đề bài. Thật vậy, do P(x) ∈ Z[x] khác hằng nên Q(x) ∈ R[x] khác đa thức 0. 68 Tạp chí Epsilon, Số 23, 06/2023 Với mọi số nguyên dương n, ta có n k ≤nk< nk + 1 ⇒ k · nk ≤ n < k · nk + k. nên n ∈ k · n k ; k · n k + 1; . . . ; k · n k + k − 1 . Do đó trong các số P k · n k , P k · n k + 1 , . . . , P k · n k + k − 1 luôn có một số là P(n). Chú ý rằng P(x) không có nghiệm nguyên nên P(n) ̸= 0, ∀n ∈ N∗. ... P(n), ∀n ∈ N∗. Bài toán được chứng minh. Từ đó suy ra Q n k Bài toán 11. Giả sử đa thức hệ số hữu tỷ f(x) bậc không bé hơn 2 và dãy số a1, a2, . . . là dãy số hữu tỉ thỏa mãn điều kiện f(an+1) = an, ∀n ≥ 1. Chứng minh rằng tồn tại số nguyên dương N để với mọi số nguyên dương n thì N an là số nguyên. Lời giải Ta chứng minh (an) bị chặn, tức là tồn tại số thực M sao cho |an| ≤ M, ∀n ≥ 1. Thật vậy, do deg f ≥ 2 nên limx→∞|f(x)| |x| = +∞, do đó tồn tại số thực M (có thể chọn M ≥ |a1|) sao cho khi |x| ≥ M thì |f(x)| ≥ |x|. Ta sẽ chứng minh |an| ≤ M, ∀n ≥ 1. Giả sử tồn tại an sao cho |an| > M, khi đó |an−1| = |f(an)| ≥ |an| > M ⇒ |an−2| = |f(an−1)| ≥ |an−1| > M · · · ⇒ |a1| > M. trái với cách chọn M, vô lý. Vậy (an) bị chặn. Do f(x) ∈ Q[x] nên ta có thể viết f(x) dưới dạng f(x) = bdxd + bd−1xd−1 + · · · + b0 c, trong đó b0, b1, . . . , bd, c ∈ Z. Giả sử a1 =rs, r, s ∈ Z. Ta đặt N = s · bd. Bằng quy nạp, ta chứng minh N an ∈ Z, ∀n ≥ 1. Thật vậy, ta có N a1 = s · bd ·rs= r · bd ∈ Z. Giả sử N an ∈ Z, với n nào đó. Xét đa thức P(x) = c · Nd bd· f x N − an . Rõ ràng P(x) ∈ Z[x] và có hệ số bậc cao nhất của x bằng 1. Chú ý rằng f(an+1) = an nên P(N an+1) = 0, hay N an+1 là nghiệm hữu tỉ của P(x), và do đó là nghiệm nguyên. Phép quy nạp hoàn tất, bài toán được giải quyết. 69 Tạp chí Epsilon, Số 23, 06/2023 Ý tưởng hay trong việc giải quyết bài toán là xây dựng đa thức hệ số nguyên có hệ số cao nhất bằng 1 và nhận N an+1 làm nghiệm. Việc xây dựng không dễ chút nào. Bài toán 12. Chứng minh rằng mọi đa thức mônic (là đa thức có hệ số cao nhất bằng 1) hệ số thực bậc n đều có thể biểu diễn bằng trung bình cộng của hai đa thức mônic hế số thực bậc n, trong đó mỗi đa thức có n nghiệm thực. Lời giải Gọi f(x) là đa thức monic bậc n hệ số thực. Nếu n = 1 thì f(x) = x + a. Xét f1(x) = x, f2(x) = x + 2a, rõ ràng mỗi đa thức có một nghiệm thực và f(x) = f1(x) + f2(x) 2. Nếu n > 1, xét đa thức g(x) = (x − 2)(x − 4)· · ·(x − 2(n − 1)) có bậc n − 1 và các đa thức P(x) = xn − k · g(x), Q(x) = −xn + k · g(x) + 2 · f(x). Ta chứng minh với k đủ lớn, P(x) và Q(x) có n nghiệm thực. Thật vậy, xét các giá trị của g(x) tại n điểm 1, 3, . . . , 2n − 1, những giá trị này đan dấu và có giá trị nhỏ nhất bằng 1 (vì nhiều nhất hai trong các thừa số có giá trị tuyệt đối bằng 1 và các thừa số còn lại có giá trị tuyệt đối lớn hơn 2). Mặt khác, tồn tại hằng số c > 0 sao cho |x|n < c và |2 · f(x) − xn| < c, với 0 ≤ x ≤ 2n. Lấy k > c thì P(x), Q(x) có giá trị đan dấu tại n điểm x = 1, 3, . . . , 2n − 1, nên P(x), Q(x) đều có ít nhất n − 1 nghiệm thực. Mà chúng là các đa thức bậc n nên mỗi đa thức đều có đủ n nghiệm thực. Mặt khác, rõ ràng chúng là mônic và trung bình cộng của chúng là f(x) nên ta thu được điều phải chứng minh. Việc xây dựng các đa thức P(x), Q(x) đảm bảo được f(x) là trung bình cộng của hai đa thức P(x)và Q(x), nhưng việc các đa thức đó có đủ n nghiệm thực cần có đa thức g(x) và hệ số k để “hiệu chỉnh”. Trên đây là một số bài toán mà quá trình giải có xây dựng đa thức, hy vọng sẽ là nguồn tài liệu tham khảo cho những bạn quan tâm đến vấn đề. Tài liệu [1] Hà Huy Khoái, Số học, NXB Giáo dục. [2] Lê Anh Vinh (chủ biên), Định hướng bồi dưỡng học sinh năng khiếu Toán, NXB Giáo dục. [3] Nguyễn Văn Mậu, Đa thức đại số và phân thức hữu tỉ, NXB Giáo dục. [4] Trần Nam Dũng (chủ biên), Các phương pháp giải toán qua các kỳ thi Olympic. [5] Tạp chí Toán học và Tuổi trẻ, NXB Giáo dục. [6] Tạp chí Pi, NXB Giáo dục. 70 Tạp chí Epsilon, Số 23, 06/2023 [7] Tạp chí Epsilon. [8] Các đề thi học sinh giỏi và chọn đội tuyển của các tỉnh. [9] Các đề thi học sinh giỏi và đề đề nghị của cuộc thi Olympic Đồng bằng Bắc Bộ. 71 Tạp chí Epsilon, Số 23, 06/2023 Câu chuyện về định lý Trung hoa về số dư Trần Nam Dũng(Trường PTNK TP HCM 72 Tạp chí Epsilon, Số 23, 06/2023 PHẦN Mở đầu I 1. Học sinh tiểu học lớp lớn hoặc THCS lớp nhỏ hầu như ai cũng biết đến bài toán tìm số cam đại loại như sau “Bà Ba đem cam ra chợ bán, khoảng trên dưới 100 quả. Khi bà xếp mớ ba thì còn thừa 2 quả, xếp mớ 5 thì thừa 3 quả còn xếp mớ 7 thì vừa đủ. Hỏi bà ba có tất cả bao nhiêu quả cam”. Một trong những cách giải thông dụng là mò từ từ. Đầu tiên, vì số cam chia 3 dư 2 nên số cam có thể là 2, 5, 8, 11, 14, · · · . Trong dãy này ta nhanh chóng tìm được số 8 chia 5 dư 3. Vậy là số 8 vừa chia 3 dư 2, vừa chia 5 dư 3. Nhưng nó chưa thỏa mãn đề bài vì 8 không chia hết cho 7. Để có thêm những số vừa chia 3 dư 2, vừa chia 5 dư 3, ta thêm vào số 8 các bội số của 15: 8, 23, 38, 53, 68, 83, 98 · · · và kiểm tra xem số nào trong chúng chia hết cho 7. Kết quả là ta tìm được số 98 thỏa mãn yêu cầu đề bài. 2. Tìm hiểu thêm một chút, ta biết bài toán này còn có tên là bài toán Hàn Tín điểm binh. Tục truyền rằng khi Hàn Tín điểm quân số, ông cho lính xếp hàng 3, hàng 5 rồi hàng 7 rồi báo cáo số dư. Từ đó ông tính chính xác quân số đến từng người. Ví dụ nếu số dư tương ứng là 2, 3, 0 thì số quân sẽ là 98. Nếu số dư, chẳng hạn là 1, 0, 2, ta cũng làm như phần 1 ở trên như sau: Xét dãy các số chia 3 dư 1 là 1, 4, 7, 10, · · · thì ta tìm được số 10 chia 3 dư 1 và chia 5 dư 0. Sau đó xuất phát từ số 10, ta cứ cộng thêm các bội của 15 vào thì được 10, 25, 40, 55, 70, 85, 100, · · · và duyệt theo số dư cho 7 thì ta được số 100 thoả mãn yêu cầu. Cách làm ổn nhưng có vẻ thủ công quá! Không lẽ mỗi lần nhận được bộ số dư ta lại mò lại từ đầu như vậy? Có cách nào để giải bài toán một cách tổng quát không? Hóa ra là có, và điều này đã được Hàn Tín đưa ra từ xưa “Lấy số dư khi chia cho 3 nhân với 70, rồi cộng với số dư của 5 nhân với 21, cộng số dư cho 7 nhân với 15 rồi cộng hoặc trừ với một bội số của 105”. Ví dụ với bộ số dư (2, 3, 0) ta được 2 × 70 + 3 × 21 + 0 × 15 = 203. Trừ đi 105 thì được 98. Với bộ số dư (1, 0, 2) thì được 1 × 70 + 0 × 21 + 2 × 15 = 100. Rất ư là lợi hại. Để nhớ quy tắc này, Hàn Tín còn đặt ra một bài thơ “Tam nhân đồng hành thất thập suy 1 73 Tạp chí Epsilon, Số 23, 06/2023 Ngũ thụ mai hoa chấp nhất chi Thất nhân đồng hành thu bán nguyệt Trừ bách trừ ngũ định vi kỳ” Bài này được Hoàng Xuân Hãn dịch thành “Ba người cùng đi, ít bảy chục Năm người cùng hàng, nhân hăm mốt Bảy người cùng hàng, nhân mười lăm Trừ trăm linh năm thì tính suốt”. 3. Công thức của Hàn Tín rất hiệu quả và thú vị. Nếu tò mò một chút, ta sẽ đặt câu hỏi: các số 70, 21, 15 có tính chất gì và được tìm ra như thế nào? Một quan sát đơn giản cho thấy số 70 chia hết cho 5, 7 và chia 3 dư 2. Số 21 chia hết cho 3, 7 và chia 5 dư 1 còn số 15 thì chia hết cho 3, 5 và chia 7 dư 1. Chính vì lẽ đó khi ta có bộ số dư là (a, b, c) thì rõ ràng số 70a + 21b + 15c sẽ có số dư khi chia cho 3, 5, 7 lần lượt là a, b, c. Và ta chỉ cần cộng hay trừ với một bội số của 105 là sẽ được số tương ứng phù hợp với yêu cầu của đề bài. Việc tìm ra các số như 70, 21, 15 cũng không có gì khó khăn. Chẳng hạn để tìm số chia hết cho 5, 7 và chia 3 dư 1 thì ta chỉ cần duyệt trong các bội số của 35: 35, 70, 105 và tất nhiên sẽ được ngay số 70. Các số 21, 15 cũng được tìm bằng cách tương tự. Phương pháp tương tự có thể được vận dụng để tìm ra các bộ số “thần bí”, khi 3, 5, 7 được thay bởi các số khác, chẳng hạn như trong bài toán sau: “Sau khi phải chui vào cái ống đồng để lên xe bắt quân kéo chạy thoát về Tàu, Thoát Hoan hú hồn nói A bát Xích kiểm điểm lại binh mã. A bát Xích cho lính xếp hàng 11 thì thấy thừa 7 người, xếp hàng 13 thì thấy thừa 5 người. Cuối cùng, ông ta cho xếp hàng 19 thì thấy thừa 6 người. Hãy cho biết số binh sĩ của Thoát Hoan lúc này biết rằng theo lịch sử thì con số đó không quá 3.000 người.” Học theo cách của Hàn Tín, ta bắt đầu đi tìm số X1 chia hết cho 13, 19 và chia 11 dư 1. Ta sẽ tìm X1 dưới dạng 13 × 19 × t = 247t. Vì 247t ≡ 5t (mod 11), nên ta nhanh chóng tìm được t = 9. Vậy X1 = 13 × 19 × 9 = 2223. Tương tự như vậy, ta tìm X2 chia hết cho 11, 19 và chia 13 dư 1. Ta sẽ tìm X2 dưới dạng 11×19×t = 209t. Vì 209t ≡ t (mod 13), nên ta nhanh chóng tìm được t = 1. Vậy X2 = 209. Cuối cùng cũng giống hoàn toàn như trên, ta tìm được X3 = 286. Áp dụng vào bài toán trên ta được đáp số là 2223 × 7 + 209 × 5 + 286 × 6 = 18322. Con số này thỏa mãn tất cả các điều kiện về số dư, nhưng quá lớn. Ta phải điều chỉnh bằng cách trừ đi một bội số của 11 × 13 × 19 = 2717. Ta trừ đi 6 lần thì được số 2020 là con số hợp lý, thỏa mãn yêu cầu đề bài. 4. Vậy định lý Trung hoa về số dư được phát biểu và chứng minh như thế nào? Rất đơn giản thôi. Cho m1, m2, . . . , mk là các số nguyên dương đôi 74 Tạp chí Epsilon, Số 23, 06/2023 một nguyên tố cùng nhau và a1, a2, . . . , ak là các số nguyên bất kỳ. Khi đó hệ phương trình đồng dư X ≡ a1 (mod m1), X ≡ a2 (mod m2), . . . , X ≡ ak (mod mk) luôn có nghiệm và hai nghiệm bất kỳ của nó sai khác nhau một bội số của M = m1m2 · · · mk. Cách chứng minh hoàn toàn tương tự như cách làm ở phần 3. Ta sẽ đi tìm các số X1, X2, . . . , Xk sao cho X1 chia hết cho m2, m3, . . . , mk và chia m1 dư 1; X2 chia hết cho m1, m3, . . . , mk và chia m2 dư 1; . . . . . . Xk chia hết cho m1, m2, . . . , mk−1 và chia mk dư 1. Và nghiệm của hệ sẽ có dạng X = a1X1 + a2X2 + · · · + akXk + M t. Đặt Mi =M mi(tích của tất cả các số mj , trừ mi). Ta chỉ cần tìm Xi dưới dạng Xi = Mi· ti. Số tiluôn tồn tại vì (Mi, mi) = 1 (đây chính là lý do ta cần đến điều kiện các mi đôi một nguyên tố cùng nhau). 5. Định lý Trung hoa về số dư có nhiều ứng dụng thú vị. Nhiều bài toán Olympic khai thác định lý này. Bài toán 1 (Olympic Phần Lan) Tồn tại hay không một đa thức P(x) nhận giá trị nguyên với mọi x nguyên, không có nghiệm nguyên nhưng với mọi số nguyên dương n đều tồn tại x sao cho P(x) chia hết cho n? Lời giải Ta chứng minh đa thức P(x) = (3x − 1)(2x − 1) thỏa mãn yêu cầu đề bài. Với n = 2k· m, ta sẽ tìm x sao cho P(x) chia hết cho 2 bằng cách chọn x sao cho(3x − 1 ≡ 0 (mod 2k) 2x − 1 ≡ 0 (mod m) Vì (3, 2k) = 1 và (2, m) = 1 nên sẽ tồn tại a và b sao cho 3a ≡ 1 (mod 2k), 2b ≡ 1 (mod m), từ đó thì hệ trên đây sẽ trở thành (x ≡ a (mod 2k) x ≡ b (mod m) và sẽ có nghiệm theo định lý Trung hoa về số dư. 75 Tạp chí Epsilon, Số 23, 06/2023 Bài toán 2 (IMO 1989) Chứng minh rằng với mọi n nguyên dương, ta luôn tìm được n số nguyên dương liên tiếp sao cho không có số nào trong chúng là lũy thừa với bậc ≥ 2 của một số nguyên tố. Lời giải Ý tưởng chính của chúng ta là: một số sẽ không phải là luỹ thừa với bậc ≥ 2 của số nguyên tố p nếu nó chia hết cho p nhưng không chia hết cho p2. Từ ý tưởng này, ta chọn các số nguyên tố phân biệt p1, p2, . . . , pn. Sau đó ta tìm X là nghiệm của hệ   X ≡ ˘1 + p1 (mod p21) X ≡ −2 + p2 (mod p22) . . . X ≡ −n + pn (mod p2n) Lúc đó X + 1, X + 2, . . . , X + n chính là n số cần tìm, vì X + i chia hết cho pi nhưng không chia hết cho p2i. 6. Quay trở lại với bài toán Hàn Tín điểm binh. Đây có phải là một ứng dụng thực tế của định lý Trung hoa về số dư? Suy nghĩ một chút, chúng ta thấy rằng Hàn Tín chẳng có lý do gì để phải điểm binh một cách phức tạp như vậy. Chỉ cần xếp hàng 7 chẳng hạn, xem được bao nhiêu hàng và dư bao nhiêu, rồi nhân lên là ra. Vậy bản chất của vấn đề là gì? Không lẽ đây chỉ là một giai thoại, một bài toán vui? Xem xét kỹ vấn đề, chúng ta thấy rằng ý đồ của Hàn Tín không phải là đếm số quân, mà là dấu số quân. Trong chiến tranh, đặc biệt là chiến tranh thời xưa, quân số là một điều tối mật, phải giữ kín. Trong quá trình báo cáo lên cho Hàn Tín, thông tin có thể bị rò rỉ bởi sẽ có thám báo khắp nơi. Bằng cách sử dụng các số dư, với trình độ của binh lính thời đó, dĩ nhiên sẽ không thể tính ra được quân số thực sự của từng nhóm và do đó sẽ không ước lượng được quân số của đội quân của Hàn Tín. Với giả định như vậy, có thể khẳng định rằng Hàn Tín là người đã đưa ra một hình thức mã hóa thông tin. Đây chưa hẳn là mật mã vì thực ra nếu lộ thông tin về các số 3, 5, 7 thì sẽ lộ ra các số 70, 21, 15 ngay. Tuy nhiên, như đã nói ở trên, với trình độ toán học chung của binh lính thì hệ mã này khá an toàn rồi. Điều này cũng như hệ mã Ceasar bên phương Tây, chỉ là dịch chuyển bảng chữ cái lên 3 đơn vị thôi cũng đủ để đối phương không đọc được các thông điệp. 7. Với trình độ toán học và khả năng tính toán như hiện nay, dĩ nhiên không thể dùng định lý Trung hoa về số dư để làm mật mã. Tuy nhiên, định lý này được dùng một cách hiệu quả trong tính toán mô-đu-la, một công cụ quan trọng của nhiều hệ thống mật mã. Trong số học mô-đu-la, chúng ta sẽ thực hiện các tính toán trong mô-đun n với n là một số rất lớn. Chúng ta sẽ cần cộng, trừ, nhân, chia, nâng lên lũy thừa . . . tất cả trong mô-đun 76 Tạp chí Epsilon, Số 23, 06/2023 này. Công việc thì khá đơn giản vì cộng trừ, nhân chia và nâng lên luỹ thừa thì ta đã học từ hồi tiểu học. Vấn đề là các số của chúng ta rất lớn, cả mô-đun n lẫn các số hạng trong các phép toán. Ý tưởng cơ bản của chúng ta là xét phân tích chính tắc 2· · · pαk 1· pα2 n = pα1 k. Ta sẽ tính toán các phép toán trong các mô-đun nhỏ pαi i(lúc này các số hạng cũng sẽ nhỏ đi khi được cắt theo mô-đun), sau đó dùng thuật toán Trung hoa số dư để phục hồi lại kết quả của phép tính cần thực hiện. Ta lấy một ví dụ đơn giản: Tìm số dư trong phép chia 22022 cho 2023. Nếu 2023 là số nguyên tố thì bài toán này quá ngon: theo định lý nhỏ Fermat, đáp số sẽ là 1. Nhưng 2023 không phải là số nguyên tố: 2023 = 7 × 172. Ta sẽ dùng sơ đồ ở trên để giải bài toán này. Trước hết ta tìm số dư khi chia 22022 cho 7. Vì 23 = 8 ≡ 1 (mod 7) nên ta dễ dàng suy ra 22022 ≡ 1 (mod 7). Để tìm số dư khi chia 22022 cho 172 = 289 ta cũng chú ý rằng φ172 = 272, nên 2272 ≡ 1 (mod 172). Từ đó do 2022 ≡ 118 (mod 272) nên 22022 ≡ 2118 (mod 172). Để tính 2118 (mod 172) ta chú ý: 118 = 64 + 32 + 16 + 4 + 2. Ta có 22 ≡ 4 (mod 172) 24 ≡ 16 (mod 172) 28 ≡ 256 (mod 172) 216 ≡ (−33)2 ≡ 222 (mod 172) 232 ≡ 2222 ≡ 154 (mod 172) 264 ≡ 1542 ≡ 18 (mod 172) Cuối cùng 2118 ≡ 4 · 16 · 222 · 154 · 18 ≡ 234 (mod 172). Từ đây, áp dụng thuật toán Trung hoa về số dư, ta tìm được 22022 chia 2023 dư 1968. Ý tưởng là như vậy, khá đơn giản nhưng đem lại nhiều hiệu quả. Ta tạm dừng câu chuyện phần 1 ở đây. Chúng ta sẽ thảo luận những vấn đề tiếp theo ở phần 2. 77 Tạp chí Epsilon, Số 23, 06/2023 PHẦN II Những vấn đề tiếp theo Trong phần 1, chúng ta khởi đầu từ các bài toán cổ đơn giản đi đến phát biểu và chứng minh định lý (cũng là thuật toán) phần dư Trung hoa ở dạng tổng quát cũng như mở đầu nói về các ứng dụng lợi hại của định lý kinh điển này. Trong phần tiếp theo chúng ta sẽ tiếp tục thảo luận về định lý Trung hoa về số dư (CRT) trên góc độ tính toán và những ứng dụng có chiều sâu hơn của định lý này. 8. Tính toán chính trong thuật toán CRT là tính xi sao cho xi ≡ 1 (mod mi) và xi ≡ 0 (modmj ) với mọi j khác i. Muốn vậy chỉ cần tìm ti sao cho Miti ≡ 1 (mod mi), trong đó Milà tích của tất cả các mj với j khác i. Vậy ti được tính như thế nào? Với các mô-đun nhỏ việc này không khó, chỉ cần duyệt vài số từ nhỏ đến lớn là ra ngay. Nhưng gặp các số lớn thì sao? Chẳng hạn với m1 = 2003, m2 = 2017, m3 = 2023 thì các được tính như thế nào? (xi = Miti). Dưới đây, ta sẽ nêu cách tính x1. Các giá trị x2 và x3 được tính tương tự. Ta cần tìm t1 sao cho m2m3t1 ≡ 1 (mod m1). Tức là 2017.2023.t1 ≡ 1 (mod 2003) ⇔ 280t1 ≡ 1 (mod 2003). Số 2003 là khá lớn nên ta khó lòng duyệt tuyến tính theo kiểu thử t1 từ 1, 2, 3, . . . Vậy ta có thể làm thế nào? Do 2003 là số nguyên tố nên 2802002 ≡ 1 (mod 2003). Như vậy ta chỉ cần chọn t1 bằng 2802001 (mod 2003) là xong. Nhưng làm sao để tính 2802001 (mod 2003)? Chẳng nhẽ nhân 2000? Thế thì có khi còn chậm hơn phép duyệt tuyến tính (mỗi lần chỉ phải cộng thêm 280 vào số dư của phép chia trước đó). Không, ta sẽ một thuật toán tốt hơn để tính 2802001 (mod 2003). Thuật toán này phần nào đã được giới thiệu ở cuối của phần 1 nhưng vẫn còn hơi thủ công. Ta sẽ trình bày phiên bản cải tiến của thuật toán nói trên, dựa vào biểu diễn nhị phân của 2001. 2001 = 11111010001(2). Để cho tiện, ta đặt a = 280. Ta lần lượt tính 1 → 1 → a → a2 → a3 → a6 → a7 → a14 → a15 → a30 → a31 → a14 → a62 → a124 → a125 → a250 → a500 → a1000 → a2000 → a2001. Như vậy, xuất phát từ 1 , ở bước i ta sẽ bình phương số đang có lên, sau đó sẽ nhân số đang có với a hoặc nhân với 1 (tức là để nguyên) tuỳ theo chữ số thứ i từ trái sang trong biểu diễn nhị phân của 2001 là 1 hay 0, rồi lại chuyển sang bước tiếp theo. 6 78 Tạp chí Epsilon, Số 23, 06/2023 Vì số 2001 trong biểu diễn nhị phân có 11 chữ số nên ta có 11 bước, mỗi bước có một hoặc hai phép nhân (bình phương hoặc nhân với a). Ở mỗi bước, vì tính theo mod 2003 nên các số hạng và kết quả đều có không quá 4 chữ Số. Một cách tổng quát, nếu ta cần tính am (mod n) (a < n, m < φ(n)) thì số phép tính cần thực hiện trong (mod n) là không quá 2len(m) trong đó len(m) là số chữ số của m trong hệ nhị phân. Vì mỗi phép nhân trong modn tốn khoảng (len(n))2 phép tính bit nên đề tính am (mod n) ta sẽ tốn khoảng 2/en(m)(len(n))2 phép tính bit. Bằng cách này ta tính được t1 = 93. Thuật toán tính am (mod n) ở trên được gọi là thuật toán bình phương liên tiếp ("repeated squarings"). 9. Như vậy, chúng ta đã tìm được một thuật toán hiệu quả để giải quyết bài toán tìm nghịch đảo của a modulo n, và từ đó giải được bài toán CRT. Tuy nhiên vì số lượng bài toán cơ bản như vậy sẽ phải thực hiện khá nhiều trong quá trình tính toán nên chúng ta cố gắng tìm cách giải “tốt” hơn, nói một cách khác là nhanh hơn cho bài toán tìm nghịch đảo modulo này. Nếu trình bày một cách ngây thơ thì cách này như sau: Ta cần tìm t và x sao cho 280t = 1 + 2003x. Từ đây thì t = 7x +1 + 43x 280, dẫn đến việc tìm x, y sao cho 1 + 43x = 280y. Từ đây lại có x = 6y +−1 + 22y 43. Ta tiếp tục tìm y, z sao cho −1 + 22y = 43z. Đến đây thì ta nhẩm nhanh được ngay z = 1, y = 2, từ đó x = 6y + z = 6 · 2 + 1 = 13, t = 7x + y = 7 · 13 + 2 = 93. Wow! Có vẻ là nhanh hơn hẳn. Nếu nhìn kỹ quá trình trên, ta sẽ thấy nó liên quan đến thuật toán Euclid để tìm ước chung của hai số 2003 và 280. Đầu tiên chia 2003 được thương là 7, dư 43, sau lại chia 280 cho 43, được thương là 6, dư 22. Lại chia 43 cho 22 được thương là 1 dư 21. Lấy 22 chia cho 21 được thương là 1 dư 1. Lấy 21 chia 1 thì không có dư. Số dư cuối cùng, 1, chính là ước chung lớn nhất của 2003 và 280. Thế còn t đã được tính thế nào từ dãy số dư nói trên? Bỏ qua phép "nhẩm nhanh"ở trên sau bước −1 + 22y = 43z ta tính tiếp Lại tìm z,u sao cho y = z +1 + 21z 22. 1 + 21z = 22u, 79 rồi z = u +u − 1 21. Tạp chí Epsilon, Số 23, 06/2023 Đến đây thì rõ ràng u = 1, z = 1. Ta thấy u = 1, z = u + 0, y = z + 1, x = 6y + z, t = 7x + y. Để đánh giá độ phức tạp của thuật toán Euclid, ta thử đánh giá số lần tối đa mà ta phải thực hiện phép chia: r0 = r1q1 + r2, r1 = r2q2 + r3, . . . Dãy này sẽ giảm nhanh, nhất là khi các số lệch nhau và thương lớn. Dãy sẽ giảm chậm nhất khi các thương qi nhỏ nhất, tức là bằng 1. Lúc này dãy sẽ giảm như dãy Fibonacci. Như thế ta ước lượng được chặn trên của số lần tối đa ta phải thực hiện phép chia là ln (r0), tức là tương đương với len (r0). Nếu chú ý rằng phép chia các số có m chữ số có độ phức tạp cũng khoảng m2thì ta ước lượng thô độ phức tạp của thuật toán Euclid là Om3 với m = len (r0), tức là không khác với cách dùng thuật toán bình phương liên tiếp ở trên. Tuy nhiên, ta nhận thấy rằng, khác với thuật toán nhân nhanh, các số mà ta thực hiện các phép toán trong thuật toán Euclid sẽ nhỏ đi rất nhanh và dĩ nhiên độ phức tạp sẽ nhỏ đi. Một đánh giá kỹ hơn sẽ cho thấy độ phức tạp để tính USCLN của a, b sẽ là O(len(a) len(b)). Việc tìm t sao cho at ≡ 1 (mod n) được thực hiện theo thuật toán tìm USCLN của n và a như ở trên, rồi phục hồi lại t, do đó độ phức tạp hoàn toàn tương tự (chỉ nhân thêm hằng số). 10. Để dễ hình dung cho việc đánh giá độ phức tạp của thuật toán Euclid mở rộng (tức là thuật toán có đầu vào là a, b đầu ra là d, s, t sao cho (a, b) = d = as + bt ta trình bày các công thức đệ quy của thuật toán này: r0 = a, r1 = b; s8 = 1, s1 = 0; t0 = 0, t1 = 1; qi = ri−1 ri ; ri+1 = ri−1 − riqi; si+1 = si−1 − s1qi; ti+1 = ti−1 − tiqi. Dễ dàng chứng minh bằng quy nạp rằng ri = asi +bti. Cho nên khi thuật toán dừng thì ta cũng tìm được s và t. Với cách trình bày này, thuật toán rất dễ để phân tích. Chú ý khi chạy thì các ri giảm dần, si và ti đan dấu và có trị tuyệt đối tăng dần. Ngoài việc tìm ra stt cho thuật toán Euclid mở rộng, s1, ti còn nhiều tính chất thú vị, cho ta nhiều ứng dụng bất ngờ. Tuy nhiên, về các ứng dụng này chúng ta sẽ đề cập đến trong một chủ đề khác, nói riêng về thuật toán Euclid và ứng dụng. Thuật toán Euclid mở rộng dễ dàng cài đặt bằng các ngôn ngữ lập trình, sử dụng vòng lặp do while hay repeat until. Nếu chạy bằng tay thì tiện nhất là dùng bảng tính Excel. 80